Head and Neck Anatomy Flashcards

1
Q

An otherwise healthy 20-year-old woman who underwent functional septorhinoplasty 3 months ago comes to the office because she says her food “does not taste normal.” Physical examination shows that the nasal airway is open and unobstructed. No functional abnormalities are noted. Which of the following is the most likely diagnosis?
A) Gustatory rhinorrhea
B) Neurapraxia of lingual nerve during the surgery
C) Olfactory dysfunction from nasal surgery
D) Taste bud alterations from prescribed antibiotics
E) Tongue injury from intubation

A

C) Olfactory dysfunction from nasal surgery

Because 80% of a meal’s flavor is aresult of olfactory input, patients often interpret a loss of smell as a loss of taste. For most patients who complain of chemosensory loss, however, the sense of taste—biologically, the sensation of salt, bitter, sweet, sour, and umami (monosodium glutamate)—which is from cranial nerve V, is intact. In a large prospective study, 7 to 9% of individuals who have undergone various types of nasal surgery experienced a decrease in olfactory function.

Gustatory rhinorrhea is an increasingly recognized complication of septoplasty and rhinoplasty that is associated with clear rhinorrhea postoperatively when the patient eats.

How well did you know this?
1
Not at all
2
3
4
5
Perfectly
2
Q

How much flavor is from olfactory input?

A

80%

How well did you know this?
1
Not at all
2
3
4
5
Perfectly
3
Q

Gustatory rhinorrhea

A

Gustatory rhinorrhea is an increasingly recognized complication of septoplasty and rhinoplasty that is associated with clear rhinorrhea postoperatively when the patient eats. This complication is not usually associated with changes in taste. It probably stems from inappropriate nerve regeneration.

How well did you know this?
1
Not at all
2
3
4
5
Perfectly
4
Q
A 67-year-old woman comes to the office because of a mass on the left side of the roof of the mouth. The patient notes that it has enlarged gradually since she first noticed it 6 months ago. Examination shows a 2-cm mass on the left hard palate and loss of sensation over the left cheek. Examination of a specimen obtained on biopsy shows adenoid cystic carcinoma. Which of the following skull-base foramina is most likely to be involved by this tumor?
A) Jugular
B) Lacerum
C) Ovale
D) Rotundum
E) Stylomastoid
A

D) Rotundum

Adenoid cystic carcinoma of the hard or soft palate is a slow-growing, insidious disease with a tendency to spread via a perineural mechanism along the palatine branchesof the maxillary division of the trigeminal nerve.

foramen rotundum (maxillary (V2) nerve

How well did you know this?
1
Not at all
2
3
4
5
Perfectly
5
Q

Adenoid cystic carcinoma

A

Adenoid cystic carcinoma of the hard or soft palate is a slow-growing, insidious disease with a tendency to spread via a perineural mechanism along the palatine branches of the maxillary division of the trigeminal nerve.

How well did you know this?
1
Not at all
2
3
4
5
Perfectly
6
Q

What passes through the stylomastoid foramen?

A

Facial nerve and stylomastoid artery

How well did you know this?
1
Not at all
2
3
4
5
Perfectly
7
Q

What passes through the foramen lacerum?

A

Internal carotid artery

How well did you know this?
1
Not at all
2
3
4
5
Perfectly
8
Q

What passes through the foramen ovale?

A

Mandibular V3 nerve, accessory meningeal artery

The lesser petrosal nerve can be transmitted as well.

How well did you know this?
1
Not at all
2
3
4
5
Perfectly
9
Q

What passes through the foramen rotundum?

A

Maxillary V2 nerve

How well did you know this?
1
Not at all
2
3
4
5
Perfectly
10
Q

What passes through the jugular foramen?

A

Glossopharyngeal IX, Vagus X, spinal accessory XI

How well did you know this?
1
Not at all
2
3
4
5
Perfectly
11
Q

Where does the internal carotid artery exit the skull?

A

Foramen lacerum

How well did you know this?
1
Not at all
2
3
4
5
Perfectly
12
Q

Where does the mandibular V3 nerve exit the skull?

A

Foramen ovale

How well did you know this?
1
Not at all
2
3
4
5
Perfectly
13
Q

Where does the maxillary V2 nerve exit the skull?

A

Foramen rotundum

How well did you know this?
1
Not at all
2
3
4
5
Perfectly
14
Q

Where does the glossopharyngeal IX nerve exit the skull?

A

Jugular foramen

How well did you know this?
1
Not at all
2
3
4
5
Perfectly
15
Q

Where does the vagus X nerve exit the skull?

A

Jugular foramen

How well did you know this?
1
Not at all
2
3
4
5
Perfectly
16
Q

Where does the spinal accessory XI nerve exit the skull?

A

Jugular foramen

How well did you know this?
1
Not at all
2
3
4
5
Perfectly
17
Q
A male newborn is evaluated because of persistent cyanosis at birth. The cyanosis is relieved by crying. Physical examination shows normal facies. Which of the following is the most likely diagnosis?
A) Choanal atresia
B) Lingual thyroid
C) Lymphatic malformation
D) Pierre Robin sequence
E) Subglottic stenosis
A

A) Choanal atresia

The patient described most likely has bilateral choanal atresia. Paradoxical cyanosis (cyanosis that is relieved by crying) is a classic symptom because newborns are obligate nose breathers. The diagnosis was traditionally confirmed by an inability to pass a catheter through the nose into the nasopharynx. CT scans are now routinely obtained and may show narrowing of the posterior nasal cavity because of medial displacement of the lateral nasal wall and pterygoid plates, as well as enlargement of the posterior vomer.

How well did you know this?
1
Not at all
2
3
4
5
Perfectly
18
Q

Paradoxical cyanosis (cyanosis that is relieved by crying) in a newborn

A

Choanal atresia

How well did you know this?
1
Not at all
2
3
4
5
Perfectly
19
Q

Presentation of choanal atresia

A

Paradoxical cyanosis (cyanosis that is relieved by crying) is a classic symptom because newborns are obligate nose breathers.

How well did you know this?
1
Not at all
2
3
4
5
Perfectly
20
Q

Paradoxical cyanosis

A

Cyanosis that is relieved by crying

How well did you know this?
1
Not at all
2
3
4
5
Perfectly
21
Q

Cause of lingual thyroid

A

Lingual thyroid results from failure of some or all of the thyroid gland to descend from the tuberculum impar of the tongue during embryonic development and presents as a posterior tongue mass that may obstruct the airway.

How well did you know this?
1
Not at all
2
3
4
5
Perfectly
22
Q

Cystic hygromas

A

Lymphatic malformations in the head and neck region

How well did you know this?
1
Not at all
2
3
4
5
Perfectly
23
Q
A 45-year-old man comes to the office for follow-up examination 3 months after undergoing surgical repair of a through-and-through laceration of the left cheek. During the procedure, an injury to the parotid duct was noted and repaired. He says he feels fine, but he now has difficulty playing the trumpet because he is unable to create sufficient air pressure in his mouth. Which of the following muscles was most likely also severed? 
A ) Buccinator 
B ) Levator labii superioris
C ) Masseter 
D ) Risorius 
E ) Zygomaticus major
A

A ) Buccinator

The buccinator muscle is the only muscle of facial expression that compresses the cheeks, which is an essential function for playing air-based instruments such as the trumpet. Both the buccinator and the orbicularis oris compress the lips, also necessary for playing trumpets

The path of the parotid duct typically leaves the parotid gland from its anterior border and courses superficially to the masseter muscle toward the mid cheek and then pierces the substance of the buccinator muscle, reaching the mucosa of the oral cavity opposite the maxillary second molar.

How well did you know this?
1
Not at all
2
3
4
5
Perfectly
24
Q

Only muscle of facial expression that compresses the cheeks

A

Buccinator

How well did you know this?
1
Not at all
2
3
4
5
Perfectly
25
Q

Course of the parotid duct

A

The path of the parotid duct typically leaves the parotid gland from its anterior border and courses superficially to the masseter muscle toward the mid cheek and then pierces the substance of the buccinator muscle, reaching the mucosa of the oral cavity opposite the maxillary second molar.

How well did you know this?
1
Not at all
2
3
4
5
Perfectly
26
Q

The buccinator muscle ordinarily contributes to:

A

The buccinator muscle ordinarily contributes to the function of forming a food bolus during mastication.

How well did you know this?
1
Not at all
2
3
4
5
Perfectly
27
Q
A 20-year-old man comes to the office because he has had paraesthesia of the anterior lateral aspect of the tongue since undergoing removal of the mandibular third molars 3 weeks ago. The most likely cause is injury to which of the following nervous structures?
A ) Chorda tympani
B ) Facial
C ) Glossopharyngeal
D ) Hypoglossal
E ) Lingual
A

E ) Lingual

General sensation of the anterior two thirds of the tongue is supplied by the lingual nerve, which is a branch of the mandibular division of the trigeminal.

How well did you know this?
1
Not at all
2
3
4
5
Perfectly
28
Q

What supplies general sensation for the anterior two thirds of the tongue?

A

General sensation of the anterior two thirds of the tongue is supplied by the lingual nerve, which is a branch of the mandibular division of the trigeminal.

How well did you know this?
1
Not at all
2
3
4
5
Perfectly
29
Q

What supplies taste in the anterior two thirds of the tongue?

A

Taste in the anterior two thirds ofthe tongue is supplied by the chorda tympani from the facial nerve. The chorda tympani joins the lingual nerve and runs anteriorly in its sheath.

How well did you know this?
1
Not at all
2
3
4
5
Perfectly
30
Q

Lingual nerve

A

General sensation of the anterior two thirds of the tongue is supplied by the lingual nerve, which is a branch of the mandibular division of the trigeminal.

How well did you know this?
1
Not at all
2
3
4
5
Perfectly
31
Q

Chorda tympani

A

Taste in the anterior two thirds ofthe tongue is supplied by the chorda tympani from the facial nerve. The chorda tympani joins the lingual nerve and runs anteriorly in its sheath.

How well did you know this?
1
Not at all
2
3
4
5
Perfectly
32
Q

The glossopharyngeal nerve supplies:

A

Mucosa of the posterior third of the tongue

How well did you know this?
1
Not at all
2
3
4
5
Perfectly
33
Q

What supplies the mucosa of the posterior third of the tongue?

A

Glossopharyngeal nerve

How well did you know this?
1
Not at all
2
3
4
5
Perfectly
34
Q

What does the hypoglossal nerve supply?

A

Motor to the tongue

How well did you know this?
1
Not at all
2
3
4
5
Perfectly
35
Q
A 26-year-old woman is being evaluated because she has had complete left-sided, flaccid facial paralysis since she awoke 3 hours ago. She also has a metallic taste in her mouth and hypersensitivity to sound. Denervation of which of the following muscles is the most likely cause of the hypersensitivity to sound?
A ) Levator palatini
B ) Stapedius
C ) Tensor tympani
D ) Tensor veli palatini
E ) Zygomaticus major
A

B ) Stapedius

The 26-year-old woman described has the typical history of Bell palsy. In patients with Bell palsy, the entire nerve is inflamed, but the maximum conduction block is either in the meatal or labyrinthine segments. Because the conduction block is proximal to thechorda tympani and stapedial nerve, patients also experience a change in taste and a decreased ability to accommodate (ie, dampen) loud noises.

How well did you know this?
1
Not at all
2
3
4
5
Perfectly
36
Q

Why do patients experience a change in taste and a decreased ability to accommodate (ie, dampen) loud noises w/ Bells palsy?

A

Because the conduction block is proximal to thechorda tympani and stapedial nerve.

How well did you know this?
1
Not at all
2
3
4
5
Perfectly
37
Q

Where is the maximum conduction block in Bell palsy?

A

In patients with Bell palsy, the entire nerve is inflamed, but the maximum conduction block is either in the meatal or labyrinthine segments.

How well did you know this?
1
Not at all
2
3
4
5
Perfectly
38
Q

Tensor tympani muscle

A

The tensor tympani muscle attaches to the malleus and is innervated by the trigeminal nerve, not the facial nerve.

How well did you know this?
1
Not at all
2
3
4
5
Perfectly
39
Q

The tensor veli palatini muscle is innervated by

A

Trigeminal nerve

How well did you know this?
1
Not at all
2
3
4
5
Perfectly
40
Q

The levator veli palatini muscle is innervated by

A

Vagus nerve

How well did you know this?
1
Not at all
2
3
4
5
Perfectly
41
Q
A 58-year-old man comes to the office for consultation regarding treatment 3 weeks after receiving a diagnosis of squamous cell cancer of the soft palate. He says he has had pain in the left ear for the past 2 months. Examination of the ear shows no abnormalities. The most likely cause of the pain is involvement of which of the following nerves?
A ) Auricular branch of the vagus (X)
B ) Auriculotemporal
C ) Great auricular
D ) Superficial temporal
E ) Vestibulocochlear (VIII)
A

A ) Auricular branch of the vagus (X)

The auricular branch of the vagus nerve (Arnold nerve) carries sensory input from the ipsilateral concha and oropharynx.

How well did you know this?
1
Not at all
2
3
4
5
Perfectly
42
Q

Arnold nerve

A

The auricular branch of the vagus nerve (Arnold nerve) carries sensory input from the ipsilateral concha and oropharynx.

How well did you know this?
1
Not at all
2
3
4
5
Perfectly
43
Q

Auriculotemporal nerve

A

The auriculotemporal nerve carries sensory information from the anterior and superior external auditory canal, and the great auricular nerve relays sensation from the lower half of the ear.

How well did you know this?
1
Not at all
2
3
4
5
Perfectly
44
Q
A 45-year-old woman comes to the office because she has had a 4-week history of a rapidly enlarging left parotid mass. On physical examination, the patient has ptosis of the left eyebrow and is unable to fully close the left eye or depress the left lower lip. The most likely cause of these findings is tumor involvement at which of the following foramina at the base of the skull?
A ) Jugular
B ) Lacerum
C ) Ovale
D ) Rotundum
E ) Stylomastoid
A

E ) Stylomastoid

A history of a progressive facial paralysis associated with a parotid mass suggests the diagnosis of a malignant parotid tumor. The temporal, zygomatic/buccal, and ramus mandibularis branches are affected, indicating that the main trunk of the facial nerve is invaded by tumor. The facial nerve exits the skull base from the stylomastoid foramen.

How well did you know this?
1
Not at all
2
3
4
5
Perfectly
45
Q
A 5-year-old boy is brought to the office because of a 10-day history of inflammation of a midline neck mass that his parents first noticed 1 year ago. Physical examination shows a 35-mm mass just inferior to the hyoid bone. Which of the following is the most likely diagnosis?
A ) Branchial cleft remnant
B ) Lingual thyroid gland
C ) Mucoepidermoid carcinoma
D ) Reactive lymph node
E ) Thyroglossal duct cyst
A

E ) Thyroglossal duct cyst

The patient described has a thyroglossal duct cyst. Thyroglossal duct cysts can form anywhere along the thyroglossal duct, which extends from the foramen cecum of the tongue to the final position of the thyroid gland in the neck, below the laryngeal cartilage.

How well did you know this?
1
Not at all
2
3
4
5
Perfectly
46
Q

Most common neck mass in children

A

Reactive lymph nodes are the most common neck mass in children. They are usually found laterally in the submandibular and jugulodigastric areas.

How well did you know this?
1
Not at all
2
3
4
5
Perfectly
47
Q

Branchial cleft remnants

A

Branchial cleft remnants (sinuses and cysts) arise from the branchial apparatus. They are also located laterally, along the anterior border of the sternocleidomastoid muscle, usually just inferior to the angle of the mandible.

How well did you know this?
1
Not at all
2
3
4
5
Perfectly
48
Q

Ectopic thyroid glands

A

Ectopic thyroid glands can be located along the course of the thyroglossal duct and are a result of failure of the thyroid to descend. Unlike thyroglossal duct cysts, they represent the only thyroid tissue present in the patient.

How well did you know this?
1
Not at all
2
3
4
5
Perfectly
49
Q
A 4-year-old child has a congenital sinus tract opening at the anterior border of the lower third of the sternocleidomastoid muscle. Which of the following nerves is most likely to be injured during surgical excision of the fistulous tract?
A ) Accessory (XI)
B ) Facial (VII)
C ) Hypoglossal (XII)
D ) Lingual
E ) Vagus (X)
A

C ) Hypoglossal (XII)

The second branchial arch descends over the third, resulting in an external opening in the lower neck. The internal opening lies at the anterior aspect of the posterior pillar of the fauces, just behind the tonsil (which is the junction between the second and third branchial arches). Usually, the fistula will follow the carotid sheath upwards before crossing the hypoglossal (XII) nerve and passing between the internal and external carotid arteries to reach the tonsillar fossa.

How well did you know this?
1
Not at all
2
3
4
5
Perfectly
50
Q

Treatment of branchial cleft cyst

A

Excision to prevent recurrence

How well did you know this?
1
Not at all
2
3
4
5
Perfectly
51
Q

Junction between the 2nd and 3rd branchial arches

A

The second branchial arch descends over the third, resulting in an external opening in the lower neck. The internal opening lies at the anterior aspect of the posterior pillar of the fauces, just behind the tonsil (which is the junction between the second and third branchial arches).

How well did you know this?
1
Not at all
2
3
4
5
Perfectly
52
Q

What nerve is at risk when excising a branchial cleft sinus or fistula?

A

Usually, the fistula will follow the carotid sheath upwards before crossing the hypoglossal (XII) nerve and passing between the internal and external carotid arteries to reach the tonsillar fossa. As a result, the hypoglossal nerve is at risk during surgery.

How well did you know this?
1
Not at all
2
3
4
5
Perfectly
53
Q
A 20-year-old man comes to the emergency department because of a deep laceration of the lower lip. Bilateral mental nerve blocks will be used to anesthetize the lip properly prior to repair. Which of the following is the most appropriate landmark for needle placement for the blocks?
A ) Mandibular second premolar
B ) Maxillary canine
C ) Oral commissure
D ) Retromolar fossa
E ) Sigmoid notch
A

A ) Mandibular second premolar

The inferior alveolar nerve enters the mandible on the medial side of the ramus about 10 mm below the sigmoid notch. It then courses through the canal closest to the buccal cortical plate in the region of the ramus, angle, and down to the third molar with an average distance of 1.8 mm ± 1 mm. The nerve then swerves away at a position of 4.1 mm + 1 mm from the buccal cortex as it passes the region of the first and second molars. As it traverses the mandibular body, it is lowest and closest to the inferior cortex (7.5 + 1.5 mm) near its exit site at the level of the first molar and second premolar via the mental foramen on the anterior surface of the mandible. The mental nerve supplies the skin of the lower lip and chin right up to the midline.

How well did you know this?
1
Not at all
2
3
4
5
Perfectly
54
Q

Where does the inferior alveolar nerve enter the mandible?

A

The inferior alveolar nerve enters the mandible on the medial side of the ramus about 10 mm below the sigmoid notch.

How well did you know this?
1
Not at all
2
3
4
5
Perfectly
55
Q

How does the inferior alveolar nerve course through the mandible?

A

After entering the mandible on the medial side of the ramus about 10 mm below the sigmoid notch, the inferior alveolar nerve then courses through the canal closest to the buccal cortical plate in the region of the ramus, angle, and down to the third molar with an average distance of 1.8 mm ± 1 mm. The nerve then swerves away at a position of 4.1 mm + 1 mm from the buccal cortex as it passes the region of the first and second molars. As it traverses the mandibular body, it is lowest and closest to the inferior cortex (7.5 + 1.5 mm) near its exit site at the level of the first molar and second premolar via the mental foramen on the anterior surface of the mandible.

How well did you know this?
1
Not at all
2
3
4
5
Perfectly
56
Q

Where does the inferior alveolar nerve exit the mandible?

A

Its exit site is at the level of the first molar and second premolar via the mental foramen on the anterior surface of the mandible.

How well did you know this?
1
Not at all
2
3
4
5
Perfectly
57
Q

What does the mental nerve supply?

A

The mental nerve supplies the skin of the lower lip and chin right up to the midline.

How well did you know this?
1
Not at all
2
3
4
5
Perfectly
58
Q

The maxillary canine can be used as a landmark for:

A

The maxillary canine may be used as a landmark for needle insertion toward the infraorbital foramen during infiltration of the infraorbital nerve.

How well did you know this?
1
Not at all
2
3
4
5
Perfectly
59
Q

The maxillary second molar is a landmark used for:

A

The maxillary second molar is a landmark typically used to locate the opening of theStensen duct

How well did you know this?
1
Not at all
2
3
4
5
Perfectly
60
Q

The retromolar fossa is the preferred location for:

A

The retromolar fossa, posterior to the mandibular third molar, is the preferred location for needle insertion to anesthetize the buccal nerve, which normally supplies sensibility to the central cheek.

How well did you know this?
1
Not at all
2
3
4
5
Perfectly
61
Q

The sigmoid notch is used as a landmark to reference:

A

The sigmoid notch is used as a landmark to reference the location of the inferior alveolar nerve.

How well did you know this?
1
Not at all
2
3
4
5
Perfectly
62
Q

The mandibular second premolar can be used as a landmark for:

A

Block of the mental nerve

How well did you know this?
1
Not at all
2
3
4
5
Perfectly
63
Q

In the temporal region, the frontal branch of the facial nerve is located within which of the following layers?
A ) Subcutaneous tissue
B ) Superficial temporal fascia
C ) Superficial layer of the deep temporal fascia
D ) Superficial temporal fat pad
E ) Deep layer of the deep temporal fascia

A

B ) Superficial temporal fascia

How well did you know this?
1
Not at all
2
3
4
5
Perfectly
64
Q

Layers from superficial to deep in the temporal area

A

(1) skin, (2) subcutaneous tissue, (3) superficial temporal fascia also known as the temporoparietal fascia, (4) superficial layer of the deep temporal fascia, (5) superficial temporal fat pad, (6) deep layer of the deep temporal fascia, (7) temporalis muscle.

How well did you know this?
1
Not at all
2
3
4
5
Perfectly
65
Q

Preventing injury to the facial nerve in a coronal approach

A

When the coronal flap is raised, as soon as the yellow superficial temporal fat pad is seen beneath the superficial layer of the deep temporal fascia, the superficial layer of the deep temporal fascia must be incised and included with the coronal flap to protect the frontal branch, which is in the superficial temporal fascia (temporoparietal fascia), one layer superficial to this.

How well did you know this?
1
Not at all
2
3
4
5
Perfectly
66
Q

In what layer is the frontal branch of the facial nerve found in the temporal region?

A

The temporal or frontal branch of the facial nerve is found within the superficial temporal or temporoparietal fascia.

How well did you know this?
1
Not at all
2
3
4
5
Perfectly
67
Q
The optic nerve passes through which of the following bones of the orbit?
A ) Ethmoid
B ) Frontal
C ) Lacrimal
D ) Maxilla
E ) Sphenoid
A

E ) Sphenoid

The lesser wing of the sphenoid forms the posterior aspect of the roof of the orbit and transmits the optic nerve and ophthalmic artery through the optic canal.

How well did you know this?
1
Not at all
2
3
4
5
Perfectly
68
Q

The superior orbital fissure transmits

A

The superior orbital fissure transmits the lacrimal nerve, frontal nerve, trochlear nerve, superior and inferior branches of the oculomotor nerve, the nasociliary nerve, and the abducens nerve.

How well did you know this?
1
Not at all
2
3
4
5
Perfectly
69
Q

What contains the superior orbital fissure

A

The greater wing of the sphenoid

How well did you know this?
1
Not at all
2
3
4
5
Perfectly
70
Q

The levator veli palatini muscle is innervated by which of the following nerves?
A ) Facial (VII)
B ) Hypoglossal (XII)
C ) Mandibular branch of the trigeminal (V)
D ) Maxillary branch of the trigeminal (V)
E ) Vagus (X)

A

E ) Vagus (X)

Embryologically, the muscle arises from the fourth pharyngeal arch and, as a result, is innervated by the pharyngeal plexus, a derivative of the vagus (X) nerve.

How well did you know this?
1
Not at all
2
3
4
5
Perfectly
71
Q

Embryological origin of the levator palatini muscle

A

Embryologically, the levator palatini muscle arises from the fourth pharyngeal arch and, as a result, is innervated by the pharyngeal plexus, a derivative of the vagus (X) nerve.

How well did you know this?
1
Not at all
2
3
4
5
Perfectly
72
Q

Ophthalmic division of the trigeminal nerve

A

The first branch, the ophthalmic division, is strictly a sensory branch.

How well did you know this?
1
Not at all
2
3
4
5
Perfectly
73
Q

Maxillary division of the trigeminal nerve

A

The second, or maxillary, division is also sensory.

How well did you know this?
1
Not at all
2
3
4
5
Perfectly
74
Q

Mandibular division of the trigeminal nerve

A

The third branch, the mandibular division, is the largest branch and is a mixed sensory and motor branch.

How well did you know this?
1
Not at all
2
3
4
5
Perfectly
75
Q

Embryological division of the mandibular nerve

A

Embryologically, the mandibular division arises from the first branchial (or mandibular) arch and provides motor function to the muscles arising from this arch.

How well did you know this?
1
Not at all
2
3
4
5
Perfectly
76
Q

Muscles innervated by the _________ division of the trigeminal nerve

A

Temporal, masseter, pterygoids, mylohyoid, tensor tympani, and the anterior belly of the digastric.

It also innervates the tensor veil palatini.

How well did you know this?
1
Not at all
2
3
4
5
Perfectly
77
Q

The anatomy of the tensor veil palatini

A

The tensor veil palatini arises from the eustachian tube and medial pterygoid plate, wraps around the hamulus, and then inserts into the midline of the soft palate.

How well did you know this?
1
Not at all
2
3
4
5
Perfectly
78
Q

What happens to the eustachian tube during swallowing?

A

During swallowing, the soft palate is tensed, opening the eustachian tube and allowing the tongue to move food posteriorly.

How well did you know this?
1
Not at all
2
3
4
5
Perfectly
79
Q
A 35-year-old man is brought to the emergency department following a motorcycle collision. Examination shows swelling and deformity of the mandible. Radiographs show a left subcondylar fracture with the proximal fragment displaced forward. Which of the following muscles is most likely involved in the displacement?
A ) Digastric
B ) Lateral pterygoid
C ) Masseter
D ) Medial pterygoid
E ) Temporalis
A

B ) Lateral pterygoid

The origin ofthe lateral pterygoid muscle (shown below) has two components: one that attaches to the temporomandibular joint capsule and disc, and one that attaches to the fovea in the neck of the mandible. Proximally (anteriorly), the superior head attaches to the greater wing of the sphenoid bone, and the inferior head to the lateral surface of the lateral pterygoid plate. Therefore, in a subcondylar fracture, the lateral pterygoid muscle is in position to draw the proximal fragment forward.

How well did you know this?
1
Not at all
2
3
4
5
Perfectly
80
Q

Anatomy of the digastric muscle

A

The digastric muscle extends from the mastoid process of the temporal bone to the hyoid bone and to the anterior mandible. It depresses the mandible (inferior movement) or elevates the hyoid.

How well did you know this?
1
Not at all
2
3
4
5
Perfectly
81
Q

Function of the digstric muscle

A

It depresses the mandible (inferior movement) or elevates the hyoid.

How well did you know this?
1
Not at all
2
3
4
5
Perfectly
82
Q

Anatomy of the lateral pterygoid

A

The origin ofthe lateral pterygoid muscle (shown below) has two components: one that attaches to the temporomandibular joint capsule and disc, and one that attaches to the fovea in the neck of the mandible. Proximally (anteriorly), the superior head attaches to the greater wing of the sphenoid bone, and the inferior head to the lateral surface of the lateral pterygoid plate. Therefore, in a subcondylar fracture, the lateral pterygoid muscle is in position to draw the proximal fragment forward.

How well did you know this?
1
Not at all
2
3
4
5
Perfectly
83
Q

Function of the medial pterygoid

A

Elevates the mandible

How well did you know this?
1
Not at all
2
3
4
5
Perfectly
84
Q

Function of the lateral pterygoid in a subcondylar fracture

A

Draws the proximal fragment forward

How well did you know this?
1
Not at all
2
3
4
5
Perfectly
85
Q

Anatomy of the masseter

A

The masseter originates in the zygomatic arch and inserts on the lateral surface of the ramus.

How well did you know this?
1
Not at all
2
3
4
5
Perfectly
86
Q

Anatomy of the temporalis muscle

A

The temporalis muscle is fan-shaped and originates from the floor of the temporal fossa and inserts in the coronoid process and anterior border of the ramus.

How well did you know this?
1
Not at all
2
3
4
5
Perfectly
87
Q

Anatomy of the medial pterygoid muscle

A

The medial pterygoid originates on the lateral pterygoid plate and the tuberosity of the maxilla and attaches to the medial surface of the angle of the mandible.

How well did you know this?
1
Not at all
2
3
4
5
Perfectly
88
Q
The nasolacrimal duct drains into the nose at which of the following anatomic locations? 
A ) Inferior concha
B ) Inferior meatus 
C ) Middle concha
D ) Middle meatus 
E ) Superior concha
A

B ) Inferior meatus

The nasolacrimal duct, which maintains the drainage of the nasolacrimal system, drains into the inferior meatus just below the inferior concha

How well did you know this?
1
Not at all
2
3
4
5
Perfectly
89
Q

Where does the nasolacrimal duct drain into?

A

The nasolacrimal duct, which maintains the drainage of the nasolacrimal system, drains into the inferior meatus just below the inferior concha.

How well did you know this?
1
Not at all
2
3
4
5
Perfectly
90
Q

The auditory tube opens into:

A

The auditory tube opens into the nasopharynx just posterior to the inferior concha.

How well did you know this?
1
Not at all
2
3
4
5
Perfectly
91
Q

The sphenoid sinus drains into:

A

The sphenoid sinus drains into the sphenoethmoid recess superior and posterior to the superior concha.

How well did you know this?
1
Not at all
2
3
4
5
Perfectly
92
Q

The posterior ethmoid air cells drain into:

A

The posterior ethmoid air cells drain into the superior meatus.

How well did you know this?
1
Not at all
2
3
4
5
Perfectly
93
Q

The nasofrontal duct drains into:

A

The middle meatus

How well did you know this?
1
Not at all
2
3
4
5
Perfectly
94
Q

The maxillary sins drains into:

A

The middle meatus

How well did you know this?
1
Not at all
2
3
4
5
Perfectly
95
Q

Where do structures drain into the nose?

A

Posterior ethmoid air cells: Superior meatus.

Sphenoid sinus drains into: sphenoethmoid recess superior and posterior to the superior concha

Nasofrontal duct: Middle meatus

Maxillary sinus: Middle meatus

Auditory tube: Nasopharynx just posterior to the inferior concha.

Nasolacrimal duct: inferior meatus just below the inferior concha

How well did you know this?
1
Not at all
2
3
4
5
Perfectly
96
Q
The mylohyoid muscle is innervated via which of the following nerves?
A ) Accessory
B ) Glossopharyngeal
C ) Hypoglossal
D ) Mandibular
E ) Maxillary
A

D ) Mandibular

How well did you know this?
1
Not at all
2
3
4
5
Perfectly
97
Q

What forms the muscular floor of the oral cavity

A

The mylohyoid muscle is a flat, triangular muscle that with its opposite forms the muscular floor of the oral cavity.

How well did you know this?
1
Not at all
2
3
4
5
Perfectly
98
Q

Anatomy of the mylohyoid muscle

A

It originates from the oblique line of the mandible from the mandibular symphysis to the last molar tooth and inserts along the body of the hyoid bone in median fibrous raphe from the mandibular symphysis to the hyoid bone.

How well did you know this?
1
Not at all
2
3
4
5
Perfectly
99
Q

Function of the mylohyoid muscle

A

The mylohyoid muscle elevates the floor of the mouth during swallowing and elevates the hyoid bone, which pushes the tongue upward during swallowing or tongue protrusion. This muscle also lowers the mandible and assists in opening the mouth, mastication, sucking, and blowing.

How well did you know this?
1
Not at all
2
3
4
5
Perfectly
100
Q

Motor innervation of the mylohyoid muscle

A

The primary motor innervation to the mylohyoid muscle is the mylohyoid branch of the inferior alveolar nerve of cranial nerve V (mandibular nerve). This nerve is a branch of the mandibular or third division of the trigeminal nerve.

How well did you know this?
1
Not at all
2
3
4
5
Perfectly
101
Q

Major nerves of the 3rd division of the trigeminal nerve

A

The two major nerves of the third division of the trigeminal nerve are the lingual nerve and the inferior alveolar nerve.

How well did you know this?
1
Not at all
2
3
4
5
Perfectly
102
Q

What is the exception for motor innervation of the tongue?

A

The exception is the palatoglossus muscle, which is supplied by cranial nerve X.
The other muscles: The hypoglossal nerve (cranial nerve XII) provides motor innervation to the tongue. This nerve suppliesall intrinsic muscles and all but one extrinsic muscle of the tongue.

How well did you know this?
1
Not at all
2
3
4
5
Perfectly
103
Q

Innervation of the tongue?

A

The hypoglossal nerve (cranial nerve XII) provides motor innervation to the tongue. This nerve suppliesall intrinsic muscles and all but one extrinsic muscle of the tongue. The exception is the palatoglossus muscle, which is supplied by cranial nerve X.

How well did you know this?
1
Not at all
2
3
4
5
Perfectly
104
Q

Innervation of the palatoglossus muscle

A

Cranial nerve X

How well did you know this?
1
Not at all
2
3
4
5
Perfectly
105
Q

The spinal accessory nerve suplies:

A

The spinal accessory nerve, which is cranial nerve XI, supplies motor innervation to the sternocleidomastoid and trapezius muscles.

How well did you know this?
1
Not at all
2
3
4
5
Perfectly
106
Q

The glossopharyngeal nerve supplies:

A

The glossopharyngeal nerve (cranial nerve IX) is a complex nerve that supplies efferent nerve innervation to the parotid gland and carries sensations to the carotid body and carotid sinus. This nerve also provides general sensation to the posterior one third of the tongue, the skin of the external ear, and the internal surface of the tympanic membrane and supplies taste for the posterior one third of the tongue.

How well did you know this?
1
Not at all
2
3
4
5
Perfectly
107
Q

What supplies efferent nerve innervation to the parotid gland?

A

The glossopharyngeal nerve (cranial nerve IX)

How well did you know this?
1
Not at all
2
3
4
5
Perfectly
108
Q

What carries sensations to the carotid body and carotid sinus.

A

The glossopharyngeal nerve (cranial nerve IX)

How well did you know this?
1
Not at all
2
3
4
5
Perfectly
109
Q

Innervation of the sternocleidomastoid muscle

A

Spinal accessory nerve

How well did you know this?
1
Not at all
2
3
4
5
Perfectly
110
Q

Innervation of the trapezius muscles

A

Spinal accessory nerve

How well did you know this?
1
Not at all
2
3
4
5
Perfectly
111
Q
The 4-month-old boy shown has congenital midline neck and mandibular anomalies, including a mild clefting of the mandible and atrophic skin in the central anterior neck. These anomalies are a result of the incomplete fusion of which of the following paired branchial arches?
A) First
B ) Second
C ) Third
D ) Fourth
E ) Fifth
A

B ) Second

How well did you know this?
1
Not at all
2
3
4
5
Perfectly
112
Q

Congenital midline cervical clefts are a result of:

A

Congenital midline cervical clefts are a result of a failure of fusion of the paired second branchial arches in the midline during embryogenesis.

How well did you know this?
1
Not at all
2
3
4
5
Perfectly
113
Q

Features of congenital midline cervical cleft

A

The most commonly reported features are a cleft of reddened, thinned tissue in the midline, a protuberance of skin superiorly, a blind epithelium-lined sinus tract caudally, and a fibrous subcutaneous cord.
The cord may limitneck movement. There is some degree of retrognathia and variable degrees of clefting of the lip and mandible.

How well did you know this?
1
Not at all
2
3
4
5
Perfectly
114
Q

Anomalies associated with midline cervical cleft

A

Anomalies that have been found in association with midline cervical cleft are thyroglossal duct cysts and ectopic bronchogenic cysts.

How well did you know this?
1
Not at all
2
3
4
5
Perfectly
115
Q

When do branchial arches develop?

A

Branchial arches begin to develop in the fourth week as neural crest cells migrate into the head and neck region. By the end of the fourth week, four pairs of branchial arches are visible externally. The fifth and sixth arches are small and not yet visible on the surface of the embryo.

How well did you know this?
1
Not at all
2
3
4
5
Perfectly
116
Q
A 70-year-old man is scheduled to undergo wedge resection of a lesion on the lower lip. Physical examination of the lower lip shows an ulcerated, well-circumscribed 5-mm lesion 1 cm medial to the right oral commissure. Which of the following foramina is the most appropriate site for instillation of anesthetic for the procedure?
A ) Inferior alveolar
B ) Infraorbital
C ) Mental
D ) Nasopalatine
E ) Sphenopalatine
A

C ) Mental

The most appropriate area for block infiltration is an intraoral blockade of the mental nerve. The inferior alveolar nerve travels through the mandible entering at the medial ramal border and exits at the mental nerve at the mental foramen at the level of the first and second bicuspid teeth, 1 cm below the alveolar ridge.

How well did you know this?
1
Not at all
2
3
4
5
Perfectly
117
Q

To block the inferior alveolar nerve:

A

Blockage of the inferior alveolar nerve foramen requires injection at its entry in the posterior mandible, or within the bony canal—an area not easily accessible.

How well did you know this?
1
Not at all
2
3
4
5
Perfectly
118
Q

Infraorbital nerve block results in:

A

Numbness of the upper lip and nasal sidewall can be accomplished with blockade of the infraorbital nerve.

How well did you know this?
1
Not at all
2
3
4
5
Perfectly
119
Q

Nasal palatine anesthetic produces:

A

Nasal palatine anesthetic produces numbness of the external nose.

How well did you know this?
1
Not at all
2
3
4
5
Perfectly
120
Q

Sphenopalatine nerve blockade produces:

A

Sphenopalatine nerve blockade produces anesthesia in the internal nose and palate.

How well did you know this?
1
Not at all
2
3
4
5
Perfectly
121
Q
The muscles of mastication are derived from which of the following branchial arches? 
A ) First 
B ) Second 
C ) Third
D ) Fourth 
E ) Fifth
A

A ) First

The muscles of mastication (temporalis, masseter, and medial and lateral pterygoids), the mylohyoid, and anterior belly ofthe digastric, and the tensors veli palatini and tympani are derived from the first branchial arch.

How well did you know this?
1
Not at all
2
3
4
5
Perfectly
122
Q

Nerve(s) derived from the 1st branchial arch

A

Mandibular Trigeminal, V2, V3

How well did you know this?
1
Not at all
2
3
4
5
Perfectly
123
Q

Nerve(s) derived from the 2nd branchial arch

A

Hyoid Facial (VII)

How well did you know this?
1
Not at all
2
3
4
5
Perfectly
124
Q

Nerve(s) derived from the 3rd branchial arch

A

Glossopharyngeal (IX)

How well did you know this?
1
Not at all
2
3
4
5
Perfectly
125
Q

Nerve(s) derived from the 4th and 6th branchial arches

A

Superior laryngeal branch of vagus (X) Recurrent laryngeal branch of vagus (X

How well did you know this?
1
Not at all
2
3
4
5
Perfectly
126
Q

Nerves vs branchial arches

A

First-Mandibular Trigeminal, V2, V3

Second-Hyoid Facial (VII)

Third-Glossopharyngeal (IX)

Fourth and Sixth-Superior laryngeal branch of vagus (X) Recurrent laryngeal branch of vagus (X)

How well did you know this?
1
Not at all
2
3
4
5
Perfectly
127
Q
A 48-year-old woman comes to the office because she has had persistent epiphora since she sustained a naso-orbital-ethmoid fracture one year ago. Jones I fluorescein dye test is performed. If the results of this test are normal, in which of the following locations is the dye most likely to be found?
(A)Inferior meatus
(B)Lower canaliculus
(C)Middle meatus
(D)Nasolacrimal duct
(E)Upper canaliculus
A

(A)Inferior meatus

In a Jones I dye test, fluorescein dye is instilled into the conjunctival sac. A cotton-tipped applicator is placed under the inferior turbinate, the site of the inferior meatus. In a positive test, dyeflows through the lacrimal system and exits at the inferior meatus.

How well did you know this?
1
Not at all
2
3
4
5
Perfectly
128
Q

% of patients with lacrimal drainage system injuries who sustain NOE trauma

A

Injuries to the lacrimal drainage system have been reported to occur in 5% to 21% of patients sustaining naso-orbito-ethmoid trauma.

How well did you know this?
1
Not at all
2
3
4
5
Perfectly
129
Q

Jones I dye test

A

In a Jones I dye test, fluorescein dye is instilled into the conjunctival sac. A cotton-tipped applicator is placed under the inferior turbinate, the site of the inferior meatus. In a positive test, dyeflows through the lacrimal system and exits at the inferior meatus. Dye is recovered on the applicator. In a negative test, no dye is recovered. This indicates an obstruction in the lacrimal drainage system.

How well did you know this?
1
Not at all
2
3
4
5
Perfectly
130
Q

Jones II dye test

A

The Jones II dye test is performed to localize the level of obstruction within the drainage system. This test is performed immediately after the Jones I test. An irrigation cannula is inserted into the punctum. Saline is irrigated through the cannula. If dye-stained fluid is obtained in the nose, thetest is positive, and a partial obstruction of the lacrimal drainage system is diagnosed. If no dye is obtained in the nose, the test is negative. If dye exits the other canaliculus, the obstruction is in the lower portion of the system. If no dye returnsthrough the other canaliculus, but dye refluxes through the same canaliculus, then the obstruction is in the upper part of the system.

How well did you know this?
1
Not at all
2
3
4
5
Perfectly
131
Q

Epiphora

A

Epiphora is the accumulation of tears that are not evacuated by the lacrimal drainage system.

How well did you know this?
1
Not at all
2
3
4
5
Perfectly
132
Q
An otherwise healthy 20-year-old woman comes to the office because she has had a painless lump on the left floor of the mouth for the past three weeks. The lump has increased in size during the past week. Physical examination shows a fluctuant, 2-cm blue mass that is not tender to palpation. Which of the following is the most likely diagnosis?
(A)Lingual thyroid
(B)Ranula
(C)Thyroglossal duct cyst
(D)Torus
(E)Vascular anomaly
A

(B)Ranula

A ranula is a mucocele or mucous extravasation phenomenon in the floor of the mouth, arising from the ducts of the sublingual or submandibular glands, often as a sequela of obstruction of the sublingual gland.

How well did you know this?
1
Not at all
2
3
4
5
Perfectly
133
Q

What is a ranula?

A

A ranula is a mucocele or mucous extravasation phenomenon in the floor of the mouth, arising from the ducts of the sublingual or submandibular glands, often as a sequela of obstruction of the sublingual gland.

How well did you know this?
1
Not at all
2
3
4
5
Perfectly
134
Q

Presentation of a ranula

A

It usually presents as a unilateral swelling of the floor of the mouth that is fluctuant and tinted blue or glossy white.
The ranula may herniate through the muscles of the floor of the mouth and present as a plunging ranula or cervical mass.

How well did you know this?
1
Not at all
2
3
4
5
Perfectly
135
Q

Treatment of a ranula

A

Treatment includes marsupialization or surgical excision including the sublingual gland.

How well did you know this?
1
Not at all
2
3
4
5
Perfectly
136
Q

Marsupialization

A

Marsupialization is the surgical technique of cutting a slit into an abscess or cyst and suturing the edges of the slit to form a continuous surface from the exterior surface to the interior surface of the cyst or abscess. Sutured in this fashion, the site remains open and can drain freely. This technique is used to treat a cyst or abscess when a single draining would not be effective and complete removal of the surrounding structure would not be desirable.

How well did you know this?
1
Not at all
2
3
4
5
Perfectly
137
Q

Lingual thyroid

A

A lingual thyroid is an uncommon condition in which the embryonic thyroid gland does not descend into the neck and presents as a firm, nontender mass at the tongue base

How well did you know this?
1
Not at all
2
3
4
5
Perfectly
138
Q

Thyroglossal duct cyst

A

A thyroglossal duct cyst represents residual epithelium-lined tracts that trace the path of descent of the thyroid and can be found anywhere in the paramedial region of the neck but are usually in the midline

How well did you know this?
1
Not at all
2
3
4
5
Perfectly
139
Q

Torus

A

A torus is an exostosis andis a slow-growing, hard mass arising from the palate or mandible. The enlargement consists of bone covered by mucosa. Excision is indicated for tori that become symptomatic or bothersome.

How well did you know this?
1
Not at all
2
3
4
5
Perfectly
140
Q
At which of the following intraosseous locations is the inferior alveolar nerve farthest from the buccal cortex?
(A)Angle
(B)First molar
(C)Ramus
(D)Second premolar
(E)Third molar
A

(B)First molar

The inferior alveolar nerve (IAN) is closest to the lingual cortex at the level of the first and second molars.
As it traverses the mandibular body, it is lowest and closest to the inferior cortex (7.5 ±1.5 mm) near its exit site at the level of the first molar and second premolar.

How well did you know this?
1
Not at all
2
3
4
5
Perfectly
141
Q

Where is the inferior alveolar nerve closest to the lingual cortex?

A

The inferior alveolar nerve (IAN) is closest to the lingual cortex at the level of the first and second molars.

How well did you know this?
1
Not at all
2
3
4
5
Perfectly
142
Q

The incidence of nerve transection during sagittal split osteotomy is _____% and usually occurs at:

A

The incidence of nerve transection during sagittal split osteotomy is 3.5% and usually occurs at the level of the third molar.

How well did you know this?
1
Not at all
2
3
4
5
Perfectly
143
Q
In the developing embryo, the stapes is formed from tissues of which of the following visceral arches?
(A)First
(B)Second
(C)Third
(D)Fourth
(E)Fifth
A

(B)Second

The first visceral arch ultimately forms the malleus, incus and the structures of the mandible. The second arch forms the stapes, the styloid and the facial musculature. The third arch forms the stylopharyngeus. The fourth, fifth and sixth arches contribute to the pharyngeal and laryngeal muscles as well as the thyroid cartilage.

How well did you know this?
1
Not at all
2
3
4
5
Perfectly
144
Q

The first visceral arch ultimately forms:

A

The first visceral arch ultimately forms the malleus, incus and the structures of the mandible.

How well did you know this?
1
Not at all
2
3
4
5
Perfectly
145
Q

The second visceral arch ultimately forms:

A

The second arch forms the stapes, the styloid and the facial musculature.

How well did you know this?
1
Not at all
2
3
4
5
Perfectly
146
Q

The third visceral arch ultimately forms:

A

The third arch forms the stylopharyngeus.

How well did you know this?
1
Not at all
2
3
4
5
Perfectly
147
Q

The 4th, 5th and 6th visceral arch ultimately forms:

A

The fourth, fifth and sixth arches contribute to the pharyngeal and laryngeal muscles as well as the thyroid cartilage.

How well did you know this?
1
Not at all
2
3
4
5
Perfectly
148
Q
A 45-year-old man has persistent maxillary sinusitis subsequent to open reduction and internal fixation of pan-facial fractures and a failed course of antibiotics. Surgical enlargement of the ostium of the sinus is planned as a drainage procedure via an endoscopic approach. Direction of the endoscope into which of the following areas within the nasal cavity is most appropriate?
(A)Inferior concha
(B)Inferior meatus
(C)Middle meatus
(D)Superior concha
(E)Superior meatus
A

(C)Middle meatus

The natural ostium of the maxillary sinus is located in the middle meatus, corresponding with the posterior portion of the ethmoidal infundibulum. Relative to the sinus, the ostium is located high and toward the back of the medial wall of the sinus, which is suboptimal for drainage by gravity. However, studies show directional movement of the mucosal cilia inside the sinus toward the ostium. Maxillary antrostomy involves enlarging the natural ostium and resecting the uncinate process of the ethmoid with the assistance of endoscopes.

How well did you know this?
1
Not at all
2
3
4
5
Perfectly
149
Q

Maxillary antrostomy

A

Maxillary antrostomy involves enlarging the natural ostium in the middle meatus and resecting the uncinate process of the ethmoid with the assistance of endoscopes.

How well did you know this?
1
Not at all
2
3
4
5
Perfectly
150
Q

The sphenopalatine foramen

A

The sphenopalatine foramen is also located in the superior meatus and connects the nasal cavity with the pterygopalatine fossa.

How well did you know this?
1
Not at all
2
3
4
5
Perfectly
151
Q
Dermoid cysts of the nasal dorsum, anterior encephaloceles, and nasal gliomas are all thought to result from failed closure of which of the following structures?
(A)Anterior fontanelle
(B)Dorsum sellae
(C)Fonticulus frontalis
(D)Foramen rotundum
(E)Metopic suture
A

(C)Fonticulus frontalis

Failed closure of the fonticulus frontalis, an embryonic fontanelle between the inferior frontal bone and nasal bone, may result in three main types of midline anterior anomalies: nasal dermal sinus, anterior cephalocele, and nasal glioma.

How well did you know this?
1
Not at all
2
3
4
5
Perfectly
152
Q

Failed closure of the fonticulus frontalis may result in:

A

Failed closure of the fonticulus frontalis, an embryonic fontanelle between the inferior frontal bone and nasal bone, may result in three main types of midline anterior anomalies: nasal dermal sinus, anterior cephalocele, and nasal glioma.

How well did you know this?
1
Not at all
2
3
4
5
Perfectly
153
Q

Fonticulus frontalis

A

The fonticulus frontalis is an embryonic fontanelle between the inferior frontal bone and nasal bone; the anterior fontanelle closes by 24 months in 96% of children.

How well did you know this?
1
Not at all
2
3
4
5
Perfectly
154
Q

Nasal dermal sinus results from:

A

Failed closure of the fonticulus frontalis

How well did you know this?
1
Not at all
2
3
4
5
Perfectly
155
Q

Anterior cephalocele results from:

A

Failed closure of the fonticulus frontalis

How well did you know this?
1
Not at all
2
3
4
5
Perfectly
156
Q

Nasal glioma results from:

A

Failed closure of the fonticulus frontalis,

How well did you know this?
1
Not at all
2
3
4
5
Perfectly
157
Q
Which of the following foramina of the middle cranial fossa hosts passage of the oculomotor (III), trochlear (IV), and abducens (VI) nerves?
(A)Optic canal
(B)Ovale
(C)Rotundum
(D)Spinosum
(E)Superior orbital fissure
A

(E)Superior orbital fissure

The foramen that allows passage of the oculomotor (III), trochlear (IV), and abducens (VI) cranial nerves is the superior orbital fissure, which is located at the orbital apex. This foramen also transmits the lacrimal, frontal, and nasociliary nerves; the sympathetic branches; the superior and inferior ophthalmic veins; the orbital branch of the middle meningeal artery; and the recurrent branch of the lacrimal artery.

How well did you know this?
1
Not at all
2
3
4
5
Perfectly
158
Q

The foramina of the orbit

A

Superior orbital fissure
The optic canal
The inferior orbital fissure

How well did you know this?
1
Not at all
2
3
4
5
Perfectly
159
Q

What passes through the optic canal?

A

The optic canal, which transmits the optic (II) nerve and the ophthalmic artery

How well did you know this?
1
Not at all
2
3
4
5
Perfectly
160
Q

What passes through the inferior orbital fissure?

A

The inferior orbital fissure, which allows passage of the infraorbital and zygomatic nerves

How well did you know this?
1
Not at all
2
3
4
5
Perfectly
161
Q
Which of the following nerves supplies sensation to the superior anterior aspect of the helix of the ear?
(A)Arnold
(B)Auriculotemporal
(C)Great auricular
(D)Greater occipital
(E)Lesser occipital
A

(B)Auriculotemporal

Sensation to the external ear is provided by the great auricular nerve, auricular branch of cranial (X) nerve (Arnold nerve), the auriculotemporal nerve, a branch of the mandibular branch of cranial (V) nerve (trigeminal nerve), and the lesser occipital nerve.

How well did you know this?
1
Not at all
2
3
4
5
Perfectly
162
Q

Sensation to the superior anterior aspect of the helix is provided by:

A

Sensation to the superior anterior aspect of the helix is provided by the auriculotemporal nerve.

How well did you know this?
1
Not at all
2
3
4
5
Perfectly
163
Q

Sensation to the ear canal is provided by:

A

The Arnold nerve provides sensation to the ear canal

How well did you know this?
1
Not at all
2
3
4
5
Perfectly
164
Q

Sensation to the posterior aspect of the external ear is provided by:

A

The posterior aspect of the external ear is provided by the lesser occipital nerve.

How well did you know this?
1
Not at all
2
3
4
5
Perfectly
165
Q

Sensation to the inferior two thirds of the external ear is provided by:

A

Most of the inferior two thirds of the external ear is provided by the great auricular nerve.

How well did you know this?
1
Not at all
2
3
4
5
Perfectly
166
Q
An otherwisehealthy 15-year-old boy is brought to the office by his parents because he has had painless swelling of the hard palate for the past year. A photograph of his mouth is shown. Physical examination shows a lesion that is solid, firm, fixed, noncompressible, nontender, and covered with healthy palatal mucosa. Which of thefollowing is the most likely diagnosis?
(A)Craniofacial fibrous dysplasia
(B)Maxillary torus
(C)Skull base glioma
(D)Squamous cell carcinoma
(E)Submucous cleft palate
A

(B)Maxillary torus

In an otherwise healthy young patient with a solid asymptomatic lesion consistent with bone on the hard palate, the most likely diagnosis is a maxillary torus or torus palatinus.

How well did you know this?
1
Not at all
2
3
4
5
Perfectly
167
Q

Maxillary and mandibular tori

A

Maxillary and mandibular tori are benign osteoblastictumors and tend to be well encapsulated, circumspect, submucosal, and expand gradually without invading adjacent structures. Torus palatinus occurs on the midline of the hard palate and requires treatment only if it becomes symptomatic or interferes with function.

How well did you know this?
1
Not at all
2
3
4
5
Perfectly
168
Q

Fibrous dysplasia (briefest overview)

A

Fibrous dysplasia is a benign tumor of bony origin that may present as a component of McCune-Albright syndrome (polyostotic fibrous dysplasia) or as a localized condition of the craniofacial skeleton, usually the upper facial skeleton.

How well did you know this?
1
Not at all
2
3
4
5
Perfectly
169
Q

External gliomas

A

Gliomas are believed to be encephaloceles that have lost their intracranial connections. External gliomas usually appear at or just lateral to the nasal root as reddish, firm, noncompressible, lobular lesions with cutaneous telangiectasia. The do not transilluminate or pulsate.

170
Q

Skull base gliomas

A

Gliomas are believed to be encephaloceles that have lost their intracranial connections. Skull base gliomas are usually intranasal, and are potentially associated with bony defects.

171
Q

Submucous cleft palates present with:

A

Submucous cleft palates present with bifid uvulas, a bony notch or defect at the edge of the hardpalate, and a zona pellucida, or thin, bluish, strip of mucosa in the midline of the soft palate secondary to diastasis of the levator muscle

172
Q
A 37-year-old man is brought to the emergency department one hour after he sustained injuries to the face in a motor vehicle collision. Physical examination shows lacerations around the bridge of the nose and mobility of the bony pyramid. Radiographs show a fracture of the nasal bones. Absence of sensation of the nasal septum and lateral nasal wall is noted. This patient has most likely sustained injuries to which of the following nerves?
(A)Anterior ethmoidal
(B)Frontal
(C)Infraorbital
(D)Infratrochlear
(E)Lacrimal
A

(A)Anterior ethmoidal

The terminal branches of the nasociliary nerve are the anterior ethmoidal nerve and the infratrochlear nerve. The former exits via a foramen of the same name, and ultimately reaches the roof of the nose, providing sensation to the septum and lateral wall. Its terminal branch, the dorsal nasal nerve, supplies innervation to the tip.

173
Q

Sensory branches of the 1st divisions of the trigeminal nerve

A

The ophthalmic branch or first division of the trigeminal nerve enters the orbit via the superior orbital fissure. The three sensory branches include the lacrimal, frontal, and nasociliary nerves.

174
Q

Nasociliary nerve

A

The nasociliary nerve courses above the optic nerve and below the superior rectus muscle. Its first branch is the posterior ethmoidal nerve; the terminal branches of the nasociliary nerve are the anterior ethmoidal nerve and the infratrochlear nerve.

175
Q

Posterior ethmoidal nerve

A

1st branch of the nasociliary nerve, providing sensation to the posterior ethmoidal sinus

176
Q

Terminal branches of the nasociliary nerve

A

Anterior ethmoidal nerve and the infratrochlear nerve

177
Q

Where does the anterior ethmoidal nerve exit?

A

The anterior ethmoidal foramen

178
Q

Anterior ethmoidal nerve termination

A

It ultimately reaches the roof of the nose, providing sensation to the septum and lateral wall. Its terminal branch, the dorsal nasal nerve, supplies innervation to the tip

179
Q
A 7-year-old boy is brought to the office for consultation regarding the congenital nerve palsy shown (right sided CN VII). The affected nerve is derived from which of the following branchial arches?
(A)First
(B)Second
(C)Third
(D)Fourth
(E)Fifth
A

(B)Second

The facial nerve (cranial nerve VII) is derived from the second branchial arch and consists primarily of motor fibers that are distributed to the muscles of facial expression.

180
Q

The facial nerve is derived from the ________ branchial arch

A

The facial nerve (cranial nerve VII) is derived from the second branchial arch and consists primarily of motor fibers that are distributed to the muscles of facial expression.

181
Q
A 76-year-old man undergoes a radial forearm flap procedure for reconstruction of a defect of the floor of the mouth resulting from excision of carcinoma. During dissection of vessels in the neck for microsurgical anastomosis, the surgeon has difficulty visualizing an appropriate recipient artery because of high bifurcation of the carotid artery. Which of the following muscles can be cut to provide better exposure of the recipient artery?
(A)Digastric
(B)Geniohyoid
(C)Omohyoid
(D)Sternohyoid
(E)Styloglossus
A

(A)Digastric

There are only two muscles that would be in the area of a high bifurcation of the carotid that may need to be cut for better exposure: posterior belly of the digastric and the stylohyoid, which span across the skull base to the hyoid. Because the stylohyoid is not mentioned, the digastric muscle is the correct choice. The microsurgeon usually divides the digastric muscle at the level of the fibrous loop for the intermediate digastric tendon. The other muscles are never dissected or divided during this procedure

182
Q

Two muscles at the area of a high carotid bifurcation that may need to be cut for better exposure

A

Posterior belly of the digastric and the stylohyoid

183
Q
Which of the following muscles of facial expression is innervated on its anterior surface?
(A) Depressor anguli oris
(B) Levator anguli oris
(C) Levator labii superioris
(D) Orbicularis oris
(E) Zygomaticus major
A

B) Levator anguli oris

The facial nerve innervates the muscles of facial expression along the posterior surface of the muscle in most cases. The exceptions to this rule include the levator anguli oris, the buccinator, and the mentalis muscle.

The levator anguli oris liesdeep to the other muscles and receives its innervation along its anterior surface.

184
Q

Which facial muscles are not innervated on their anterior surface?

A

The levator anguli oris, the buccinator, and the mentalis muscle.

185
Q
Formation of the primary palate begins during which of the following weeks of gestation?
(A)First
(B)Third
(C)Fifth
(D)Eighth
(E)Eleventh
A

(C)Fifth

Formation of the primary palate begins during the fifth week of gestation and is completed by the end of the sixth week of gestation. Anatomically, it is located anterior to the incisive foramen. The premaxilla only contains the central and lateral incisors.

186
Q

Timeline for formation of the primary palate

A

Formation of the primary palate begins during the fifth week of gestation and is completed by the end of the sixth week of gestation.

187
Q

Which fascia envelops the trapezius?

A

Superficial fascia

188
Q

What fascia envelops the anterior scalene muscle?

A

Prevertebral fascia

189
Q

The superficial fascia in the neck envelops the:

A

The superficial fascia envelops the sternocleidomastoid, the trapezius, and the suprahyoid muscles.

190
Q

The prevertebral fascia in the neck incorporates:

A

The prevertebral fascia incorporates all of the scalene and paravertebral muscles.

191
Q

The pretracheal fascia incorporates:

A

The pretracheal fascia incorporates the thyroid and trachea

192
Q
Which of thefollowing muscles of the mouth is innervated by the mandibular branch of the facial (VII) nerve?
(A)Buccinator
(B)Depressor anguli oris
(C)Levator anguli oris
(D)Orbicularis oris
(E)Risorius
A

(B)Depressor anguli oris

The other mimetic muscles innervated by the mandibular branch of the facial nerve include the depressor labii inferioris and the mentalis muscle. These muscles control movement of the lower lip to either pull it down and laterally or to protrude it.

193
Q

Muscles innervated by the mandibular branch of the facial nerve

A

The other muscles innervated by the mandibular branch of the facial nerve include the depressor anguli oris, the depressor labii inferioris and the mentalis muscle. These muscles control movement of the lower lip to either pull it down and laterally or to protrude it.

194
Q
Which of the following structures passes through the pterygomaxillary fissure?
(A) Mandibular artery
(B) Mandibular nerve
(C) Maxillary artery
(D) Maxillary nerve
A

(C) Maxillary artery

195
Q

The pterygomaxillary fissure connects the small pterygopalatine fossa with:

A

The pterygomaxillary fissure connects the small pterygopalatine fossa with the much larger infratemporal fossa.

196
Q

The pterygomaxillary fissure transmits:

A

The pterygomaxillary fissure transmits the terminal branches of the maxillary artery and veins.

197
Q

Borders of the pterygomaxillary fissure

A

The posterior border of this opening is the anterior aspect of the pterygoid plates. The anterior border is the posterior aspect of the maxilla, and the superior border is the pterygopalatine fossa and sphenoid bone and the orbital process of the palatine bone.

198
Q

Appropriate care must be taken during separation of the face from the pterygoid plates to avoid injury to:

A

Appropriate care must be taken during separation of the face from the pterygoid plates to avoid injury to the maxillary artery and veins.

199
Q
A 56-year-old woman undergoes reconstruction of the mandible using a free fibular flap. During the procedure, the microsurgeon dissects the external carotid artery; however, the superior thyroid artery cannot be located for anastomosis. Which of the following branches of the external carotid artery is the next distal vessel that would be suitable for anastomosis?
(A) Lingual
(B) Occipital
(C) Posterior auricular
(D) Maxillary
A

(A) Lingual

The branches of the external carotid artery, in order from proximal to distal, are as follows: superior thyroid, ascending pharyngeal, lingual, occipital, facial, posterior auricular, and maxillary arteries.

200
Q

The branches of the external carotid artery, in order from proximal to distal:

A
The branches of the external carotid artery, in order from proximal to distal: 
Superior thyroid
Ascending pharyngeal
Lingual
Occipital
Facial
Posterior auricular
Maxillary
201
Q
A 64-year-old man comes to the office because he has numbness of the left medial cheek and infraorbital area. Physical examination shows a mass of the hard palate. Incisional biopsy shows adenoid cystic carcinoma. Further evaluation is most likely to show tumor involvement of which of the following foramina at the base of the skull?
(A) Anterior ethmoid foramen
(B) Foramen ovale
(C) Foramen rotundum
(D) Jugular foramen
(E) Stylomastoid foramen
A

(C) Foramen rotundum

Numbness in the left medial cheek and left infraorbital area suggest that the tumor has invaded the infraorbital nerve, which exits the middle cranial fossa from the foramen rotundum.

202
Q

% of patients with adenoid cystic carcinoma who have perineural spread

A

Approximately 80% of patients with adenoid cystic carcinoma have perineural spread.

203
Q
Which of the following muscles elevates the mandible?
(A) Digastric 
(B) Genioglossus 
(C) Geniohyoid 
(D) Medial pterygoid 
(E) Mylohyoid
A

(D) Medial pterygoid

The masseter, temporalis, and medial pterygoid muscles elevate the mandible, and the geniohyoid, genioglossus, mylohyoid, and digastric muscles tend to depress the mandible.

The lateral pterygoid muscle inserts into the capsule of the temporomandibular joint and tends to remain attached and pull the condyle head medial when there is a high fracture.

204
Q

Which muscles elevate the mandible?

A

The masseter, temporalis, and medial pterygoid muscles elevate the mandible.

205
Q

Which muscles tend to depress the mandible?

A

The geniohyoid, genioglossus, mylohyoid, and digastric muscles tend to depress the mandible.

206
Q
Which of the following muscles can function to close off the oral cavity from the oropharynx?
(A) Levator veli palatini
(B) Musculus uvulae
(C) Palatoglossus
(D) Palatopharyngeus 
(E) Tensor veli palatini
A

(C) Palatoglossus

The palatoglossus is a paired muscle that elevates the posterior tongue and pulls it against the soft palate, separating the oral cavity from the oropharynx. The muscle attaches to the side of the tongue on one end and the palatine aponeurosis on the other.

207
Q

Function / anatomy of the palatoglossus

A

The palatoglossus is a paired muscle that elevates the posterior tongue and pulls it against the soft palate, separating the oral cavity from the oropharynx. The muscle attaches to the side of the tongue on one end and the palatine aponeurosis on the other.

208
Q

Function / anatomy of the levator veli palatini

A

The levator veli palatini elevates the soft palate during swallowing and yawning. It attaches superiorly to the cartilage of the auditory tube and the petrous part of the temporal bone and inferiorly to the palatine aponeurosis

209
Q

Anatomy / function of the palatopharyngeus muscle

A

The palatopharyngeus muscle is part of the palatopharyngeal arch, attaching to the lateral wall of the pharynx, the hard palate, and the palatine aponeurosis. During swallowing, it tenses the soft palate while pulling the walls of the pharynx superiorly, medially, and anteriorly, effectively closing off the nasopharynx from the oropharynx

210
Q
A 6-year-old boy who underwent repair of a unilateral cleft lip and palate in infancy is brought to the office by his parents because he has had a sensation of numbness in the left anterior mobile tongue for the past two days, since he underwent repair of a palatal fistula during orotracheal intubation. During that procedure, the oral cavity and palate were retracted for two hours with a Dingman retractor. Which of the following nerves was most likely damaged during the procedure?
(A) Glossopharyngeal (IX)
(B) Lingual
(C) Mental
(D) Recurrent laryngeal
(E) Superior laryngeal
A

(B) Lingual

The lingual nerve provides sensation to the anterior two thirds of the tongue. Injury to the lingual nerve is a rare complication of airway manipulation. Virtually all equipment associated with airway management and/or oral cavity retraction has been implicated in its damage.

211
Q

Etiology of operative damage to the lingual nerve

A

The etiology of lingual nerve damage is likely compression of the nerve in the floor of mouth against the mandible caused by forceful compression of the floor of mouth.

212
Q

The mental nerve exits the mental foramen:

A

The mental nerve exits the mental foramen just below the first or second premolar and supplies sensation to the lower lip and chin.

213
Q

The superior laryngeal nerve provides:

A

The superior laryngeal nerve provides sensory innervation to the supraglottis.

214
Q

The glossopharyngeal nerve provides:

A

The glossopharyngeal nerve provides sensory innervation and taste sensation to the base of the tongue.

215
Q

The recurrent laryngeal nerve provides:

A

The recurrent laryngeal nerve provides motor innervation to the laryngeal musculature.

216
Q

Which of the following best describes the pathway of the accessory (XI) nerve after it enters the posterior triangle of the neck?
(A) Within the subcutaneous fat
(B) Within the carotid sheath
(C) Within the inferior portion of the sternocleidomastoid muscle
(D) Deep to the investing fascia
(E) Deep to the prevertebral fascia

A

(D) Deep to the investing fascia

CN XI enters the posterior triangle of the neck passing obliquely across the floor over the levator scapulae just deep to the investing fascia and superficial to the prevertebral fascia.

217
Q

The accessory nerve exits the skull at:

It enters the neck between:

A

The accessory nerve descends through the jugular foramen into the neck between the internal carotid and internal jugular vein.

218
Q

After entering the neck, CN XI crosses the ________ and is joined by ________

A

It crosses the internal jugular vein and is joined by fibers from the ventral ramus of C2

219
Q

What does CN XI give off in the neck?

A

It gives off a motor branch to the sternocleidomastoid muscle before passing deep to or through this muscle. It emerges on the posterior border of the sternocleidomastoid muscle always in the upper half of this muscle.

220
Q

Where does CN XI enter the posterior triangle of the neck?

A

It enters the posterior triangle of the neck passing obliquely across the floor over the levator scapulae just deep to the investing fascia and superficial to the prevertebral fascia. It supplies motor fibers to the trapezius.

221
Q
The papilla of the parotid duct is most commonly located adjacent to which of thefollowing maxillary teeth?
(A) First bicuspid
(B) Second bicuspid
(C) First molar
(D) Second molar
(E) Third molar
A

(D) Second molar

The papilla of the parotid is located most frequently adjacent to the maxillary second molaras it empties the secretions of the parotid gland via Stensen’s duct.

222
Q

Which of the following structures travels through the foramen rotundum?
(A) First division of the trigeminal (V) nerve
(B) Second division of the trigeminal (V) nerve
(C) Third division of the trigeminal (V) nerve
(D) Optic (II) nerve
(E) Middle meningeal artery

A

(B) Second division of the trigeminal (V) nerve

The maxillary division of the trigeminal nerve travels through the foramen rotundum. The ophthalmicdivision of the trigeminal nerve travels through the superior orbital fissure. The mandibular division of the trigeminal nerve travels through the foramen ovale. The foramen spinosum carries the middle meningeal artery. The optic nerve travels through theoptic foramen.

223
Q

Through what foramen do the trigeminal nerves pass?

A

Ophthalmic: Superior orbital fissure
Maxillary: Foramen rotundum
Mandibular: Foramen ovale

224
Q
Which of the following muscles is included in a facial artery musculomucosal flap?
(A) Buccinator
(B) Depressor anguli oris
(C) Levator labii superioris
(D) Orbicularis oris
(E) Zygomaticus major
A

(A) Buccinator

Because the buccinator muscle is sandwiched between the facial artery and the oral mucosa, it must be included in a facial artery musculomucosal flap. The buccinator muscle originates from the pterygomandibular raphe and inserts into the orbicularis oris muscle and mucosa of the lateral lip elements

225
Q

Origin and insertion of the buccinator muscle

A

The buccinator muscle originates from the pterygomandibular raphe and inserts into the orbicularis oris muscle and mucosa of the lateral lip elements.

226
Q

What muscle must be included in the facial artery musculomucosal flap?

A

Because the buccinator muscle is sandwiched between the facial artery and the oral mucosa, it must be included in a facial artery musculomucosal flap.

227
Q
Which of the following muscles of the palate is innervated predominantly by the trigeminal (V) nerve? 
(A) Levator veli palatini
(B) Musculus uvula
(C) Palatoglossus
(D) Palatopharyngeus
(E) Tensor veli palatini
A

(E) Tensor veli palatini

The tensor veli palatini muscle is innervated predominantly by the mandibular branch of the trigeminal nerve (V3). In contrast, the other palatal muscles, including the levator veli palatini, musculus uvula, palatoglossus, and palatopharyngeus, are innervated predominantly by branches of the pharyngeal plexus of the vagus (X) nerve.

228
Q

The tensor veli palatini is innervated by:

A

The mandibular branch of the trigeminal nerve (V3)

229
Q

The levator veli palatini is innervated by:

A

The pharyngeal plexus of the vagus (X) nerve.

230
Q

The uvula is innervated by:

A

The pharyngeal plexus of the vagus (X) nerve.

231
Q

The palatoglossus is innervated by:

A

The pharyngeal plexus of the vagus (X) nerve.

232
Q

The palatopharyngeus is innervated by:

A

The pharyngeal plexus of the vagus (X) nerve.

233
Q
A 65-year-old man undergoes operative removal of a basal cell carcinoma at the junction of the upper cheek and temporal region, followed by coverage of the resultant 10 x 5-cm defect with a submental myocutaneous flap. This flap derives its blood supply from a branch of which of the following arteries? 
(A) Facial
(B) Inferior thyroid
(C) Lingual
(D) Superior thyroid
(E) Transverse cervical
A

(A) Facial

The submental flap is a myocutaneous flap that is useful in head and neck reconstruction. The flap is elevated below the level of the platysma muscle and includes the submental artery and vein, which are direct branches of the facial artery and vein. The flap can be transposed to cover defects in the lower and central thirds of the face and into the inferior aspect of the upper third of the face.

234
Q

Submental flap: usage

A

The flap can be transposed tocover defects in the lower and central thirds of the face and into the inferior aspect of the upper third of the face.

235
Q

Elevation of the submental flap:

A

The flap is elevated below the level of the platysma muscle and includes the submental artery and vein, which are direct branches of the facial artery and vein.

236
Q

Submental artery:

A

The submental artery is a consistent branch of the facial artery and gives off one or two cutaneous perforators to the submental skin. The submental artery runs in relation to the anterior belly of the diagastric muscle.

237
Q

The superior thyroid and interior thyroid arteries supply:

A

The superior thyroid and inferior thyroid arteries provide the blood supply to the thyroid gland.

238
Q

The transverse cervical artery supplies:

A

The transverse cervical artery gives off a descending branch, which provides the blood supply to the trapezius muscle.

239
Q
The parotid (Stensen) duct opens into the mouth at which of the following levels?
(A) Lower first molar
(B) Lower second molar
(C) Upper first molar
(D) Upper second molar
(E) Upper third molar
A

(D) Upper second molar

The parotid (Stensen) duct opens into the mouth at the level of the upper second molar. It is about 4 to 7 cm long, extends from the anterior border of the superficial lobe of the parotid gland, and is frequently accompanied by accessory parotid tissue.

240
Q

Course of Stensen’s duct

A

From the border of the superficial lobe of the parotid, the duct runs anteriorly over the masseter muscle about halfway between the zygomatic arch and the angle of the mouth. It turns medially beyond the anterior border of the masseter muscle, pierces the buccinator muscle, and opens into the mouth through the parotid papilla at about the level of the upper second molar.

241
Q

What nerves may cross the parotid duct

A

The zygomatic and buccal branches of the facial (VII) nerve may cross superficially over the parotid duct

242
Q

Vertical vs horizontal incisions over the parotid

A

Vertical incisions at the anterior border of the parotid gland maycause injury to the parotid duct or branches of the facial (VII) nerve. Horizontal incisions are less likely to damage these structures.

243
Q

Anatomical considerations when draining a parotid abscess

A

Horizontal incisions are less likely to damage the parotid duct or branches of the facial (VII) nerve.
Because the parotid gland is divided by fascial septa that extend from its capsule, a parotid abscess may be compartmentalized. Therefore, multiple horizontal incisions may be needed for adequate drainage.

244
Q

Clinical presentation of a parotid abscess

A

Also, a parotid abscess may not be fluctuant because of the septa. Pitting edema over the parotid gland usually indicates an abscess

245
Q
Which of the following muscles contributes to Passavant’s ridge?
(A) Middle pharyngeal constrictor
(B) Styloglossus
(C) Stylopharyngeus
(D) Superior pharyngeal constrictor
(E) Tensor veli palatini
A

(D) Superior pharyngeal constrictor

During gagging, forceful contraction of the superior pharyngeal constrictor and levator palatini muscles may produce Passavant’s ridge, which is a bulge on the posterior pharynx above the arch of the atlas. In patients with velopharyngeal incompetence (VPI), this ridge may be activated as a compensatory mechanism and may serve as a reference point during surgery to correct VPI.

246
Q

Passavant’s ridge

A

During gagging, forceful contraction of the superior pharyngeal constrictor and levator palatini muscles may produce Passavant’s ridge, which is a bulge on the posterior pharynx above the arch of the atlas.

247
Q

Passavant’s ridge and VPI

A

In patients with velopharyngeal incompetence (VPI), the ridge may be activated as a compensatory mechanism and may serve as a reference point during surgery to correct VPI.

248
Q

Anatomy of the middle pharyngeal constrictor

A

The middle pharyngeal constrictor muscle arises from the hyoid bone and is inferior to the superior pharyngeal constrictor.

249
Q
Actions of the levator veli palatini muscle include retracting the soft palate and which of the following?
(A) Closing the eustachian tube 
(B) Drawing the uvula anteriorly
(C) Narrowing the faucial isthmus
(D) Opening the eustachian tube
(E) Widening the faucial isthmus
A

(D) Opening the eustachian tube

The soft palate consists of the levator veli palatini muscle as well as the muscles of the uvula and the palatopharyngeus, palatoglossus, and tensor veli palatini muscles. The levator veli palatini muscle raises and retracts the soft palate to bring it in touch with the posterior pharyngeal wall. It also opens the eustachian tube. The uvular muscles draw the uvula superiorly and anteriorly. The palatopharyngeus and palatoglossus muscles lower the soft palate and narrow the faucial isthmus. The tensor veli palatini muscle tenses and lowers the soft palate and opens the eustachian tube.

250
Q

What opens the eustachian tube?

A

The levator veli palitini and the tensor veli palatini

251
Q

Isthmus

A

Narrowest part

In geography it is a narrow strip of land connecting two larger pieces

252
Q

Why is it difficult to approximate the tensor veli palatini?

A

In the repair of a cleft of the soft palate, the tensor veli palatini muscle is difficult to approximate because the tendon runs overthe hamulus. To avoid separation of the tensor tendon, it can be dissected off the hamulus by fracturing the hamulus.

253
Q

Challenge in approximating the tensor veli palatini vs the levator veli palatini?

A

In the repair of a cleft of the soft palate, the tensor veli palatini muscle is difficult to approximate because the tendon runs overthe hamulus. In contrast, the levator veli palatini muscle can be easily approximated.

254
Q
Theforamen rotundum is situated within which of the following structures?
(A) Anterior cranial fossa
(B) Cribriform plate
(C) Lesser petrosal sinus
(D) Orbit
(E) Sphenoid bone
A

(E) Sphenoid bone

The foramen rotundum is situated within the sphenoid bone in the middle cranial fossa, not on a lesser wing of the sphenoid bone, which is located in the anterior cranial fossa.

The foramen rotundum transmits the maxillary branch (V2) of the trigeminal (V) nerve from the middle cranial fossa into thepterygopalatine fossa.

255
Q

The foramen rotundum transmits the _________ from the ___________ into the ___________

A

The foramen rotundum transmits the maxillary branch (V2) of the trigeminal (V) nerve from the middle cranial fossa into the pterygopalatine fossa.

256
Q

The anterior cranial fossa contains which bones?

A

The anterior cranial fossa contains the frontal and ethmoid bones and the lesser wings of the sphenoid bone.

257
Q

The foramina of the anterior cranial fossa include:

A

The foramina of the anterior cranial fossa include the foramen cecum, foramina of the cribriform plate, and the anterior and posterior ethmoidal canals.

258
Q

The foramen rotundum is situated within the _________ bone in the __________ fossa

A

The foramen rotundum is situated within the sphenoid bone in the middle cranial fossa.

259
Q

The cribriform plate comes in contact with the __________ on one side and with the _________ on the other.

A

The cribriform plate (the horizontal component of the ethmoid bone) comes in contact with the anterior cranial fossa on one side and with the nasal cavity on the other. The cribriform plate contains multiple foramina for the olfactory (I) nerves

260
Q

The lesser petrosal sinus is located in the ________________ in the ____________ fossa

A

The lesser petrosal sinus is located in the anterior wall of the petrous part of the temporal bone in the middle cranial fossa. It transmits the lesser petrosal nerve.

261
Q

The lesser petrosal sinus transmits what?

A

It transmits the lesser petrosal nerve.

262
Q
Which of the following muscles of facial animation is innervated via its superficial surface?
(A) Buccinator
(B) Depressor anguli oris
(C) Orbicularis oculi
(D) Orbicularis oris
(E) Zygomaticus major
A

(A) Buccinator

263
Q

Why is dissection in the subcutaneous plane of the face safe?

A

Dissection within the subcutaneous plane is safe because this plane does not contain the facial (VII) nerve.

264
Q

Why is dissection superficial to the SMAS safe?

A

Dissection superficial to the SMAS layer is safe because the muscles of facial animation are innervated from below, except for the buccinator, mentalis, and levator anguli oris muscles, which are innervated via their superficial surfaces.

265
Q

Important when dissecting below the SMAS plane

A

The depressor anguli oris, orbicularis oculi, orbicularis oris, and zygomaticus major are all innervated from their deep surfaces. When dissecting below the SMAS plane, the zygomaticus major muscle serves as an important landmark. In this plane, dissection should proceed more superficially to preserve innervation of the zygomaticus major muscle.

266
Q
A neonate has a reddish 1.5-cm mass of the nasal root with overlying cutaneous telangiectasias. A photograph is shown above. On physical examination, the mass is firm, noncompressible, and nonpulsatile. It does not transilluminate or change with Valsalva maneuver. Which of the following is the most likely diagnosis?
(A) Dermoid cyst
(B) Encephalocele
(C) Glioma
(D) Hemangioma
(E) Lipoma
A

(C) Glioma

The findings in this neonate are consistent with a glioma. Nasal gliomas are thought to originate as encephaloceles but fail to maintain their intracranial connections. They may be external, internal, or a combination of both. External gliomas typically appear at or just lateral to the nasal root. They are reddish, firm, noncompressible, lobular lesions that exhibit telangiectasias of the overlying skin, but do not transilluminate or pulsate. Bony defects, intracranial connections, and cerebrospinal fluid leakage occur only rarely. Histologic evaluation shows astrocytic neuroglial cells and fibrous and vascular connective tissue that is covered with skin or nasal mucosa.

Lipomas are soft, skin colored, compressible lesions that do not have cutaneous telangiectasias and do not transilluminate or pulsate. They may appear at the nasal root, but are not predisposed to that location.

267
Q

Presentation of external glioma

A

External gliomas typically appear at or just lateral to the nasal root. They are reddish, firm, noncompressible, lobular lesions that exhibit telangiectasias of the overlying skin, but do not transilluminate or pulsate.

268
Q

How are nasal gliomas thought to arise?

A

Nasal gliomas are thought to originate as encephaloceles but fail to maintain their intracranial connections. .

269
Q

Rare complications of nasal gliomas

A

Bony defects, intracranial connections, and cerebrospinal fluid leakage occur only rarely.

270
Q

Histological evaluation of nasal glioma

A

Histologic evaluation shows astrocytic neuroglial cells and fibrous and vascular connective tissue that is covered with skin or nasal mucosa.

271
Q

Origin of nasal dermoid cysts

A

A nasal dermoid cyst arises from a dermoid sinus, which is a cutaneous inward passage lined with stratified squamous epithelium.

272
Q

Presentation of nasal dermoid cyst

A

These masses can also be external or internal. An external nasal dermoid is a firm, noncompressible, nonpulsatile lesion that does not transilluminate and may be lobulated.

273
Q

Rare complications of nasal dermoid cysts

A

Although bony defects are infrequent, cerebrospinal fluid leakage and meningitis may occur.

274
Q

What are nasal dermoid cysts composed of?

A

Nasal dermoid cysts are derived from ectoderm and mesoderm, lined with squamous epithelium, and contain specialized adnexal structures such as hair follicles, pilosebaceous glands, and smooth muscles.

275
Q

What is an encephalocele?

A

Encephaloceles involve herniation of cranial tissue through a skull defect. They may be classified as meningoceles (containing meninges only), meningoencephaloceles (containing meninges and brain), or meningoencephalocystoceles (containing meninges, brain, and part of the ventricular system).

276
Q

Presentation of external encephalocele

A

External, or sincipital, encephaloceles are soft, bluish, compressible, pulsatile masses that are located at the nasal root and transilluminate. They typically enlarge with crying and Valsalva maneuver

277
Q

Meningoceles

A

Containing meninges only

278
Q

Meningoencephaloceles

A

Containing meninges and brain

279
Q

Meningoencephalocystoceles

A

Containing meninges, brain, and part of the ventricular system

280
Q
A 25-year-old woman is scheduled to undergo lip augmentation using dermal allograft. Blockade of the infraorbital and mental nerves will be performed bilaterally. In this patient, direct infiltration of local anesthetic is most likely to be required at which of the following sites?
(A) Central upper vermillion
(B) Central lower vermillion
(C) Commissures 
(D) Cupid’s bow
(E) Philtrum
A

(C) Commissures

Because blockade of the infraorbital and mental nerves alone does not ensure adequate anesthesia of the oral commissures, direct infiltration of local anesthetic into the commissures is likely to be necessary. Although a portion of the sensibility of this region is supplied by contributions from the infraorbital and mental nerves, the greatest contributor is the buccal nerve, which is derived from the mandibular branch of the trigeminal nerve (V3). Although the infraorbital nerve may reach the upper half of the commissure and extend to 1.5 cm laterally, the border of this region follows an upward curve away from the commissure.

281
Q

Ensuring anesthesia of the lip: blockade of the infraorbital and mental nerve will miss

A

Will not anesthetize the commissure

Although the infraorbital nerve may reach the upper half of the commissure and extend to 1.5 cm laterally, the border of this region follows an upward curve away from the commissure

282
Q

Bilateral blockade of the infraorbital nerve vs the lip:

A

Bilateral blockade of the infraorbital nerve in the midline provides complete anesthesia to all central components of the upper lip, including the vermillion, cupid’s bow, and philtrum.

283
Q

Bilateral blockade of the mental nerve vs the lip:

A

Bilateral mental nerve blocks effectively anesthetize the central section of the lower lip because the regions supplied by these nerves meet in the midline.

284
Q
Which of the following nerves supplies sensory innervation to the buccal mucosa?
(A) Trigeminal (V) nerve
(B) Facial (VII) nerve
(C) Glossopharyngeal (IX) nerve
(D) Vagus (X) nerve
(E) Lingual nerve
A

(A) Trigeminal (V) nerve

The buccal branch of the trigeminal (V) nerve provides sensation to the buccal mucosa. It is important for the surgeon to know the anatomy of this nerve branch to plan and perform neurotized free flap reconstruction and reinnervation of the intraoral cavity.

The buccal branch of the facial (VII) nerve innervates the muscles surrounding the buccal mucosa.

285
Q

A 38-year-old woman sustains an injury to the auriculotemporal nerve during superficial parotidectomy for removal of a mixed tumor. Which of the following is the most likely postoperative finding?
(A) Numbness of the concha, helix, lobule, and temporal skin
(B) Numbness of the tragus, external auditory canal, and temporal skin
(C) Numbness of the entire pinna and paralysis of the temporalis muscle
(D) Paralysis of the auricularis anterior, superior, and posterior muscles
(E) Paralysis of the temporalis muscle

A

(B) Numbness of the tragus, external auditory canal, and temporal skin

286
Q

The auriculotemporal nerve is a branch of:

A

The auriculotemporal nerve is a branch of the mandibular division of the trigeminal nerve (V3).

287
Q

What does the auriculotemporal nerve innervate?

A

Cutaneous sensory innervation to the tragus, external acoustic meatus, tympanic membrane, and temporal region. Minor branches of the auriculotemporal nerve convey secretomotor fibers to the parotid gland and articular fibers to the temporomandibular joint.

288
Q

Relation of the auriculotemporal nerve to the middle meningeal artery

A

The fascicles of this nerve divide soon after the nerve originates from V3 to allow for passage of the middle meningeal artery; it then courses between the sphenomandibular ligament and the neck of the condyle.

289
Q

Relation of the auriculotemporal nerve to the parotid

A

After coursing between the sphenomandibular ligament and the neck of the condyle, It emerges from behind the temporomandibular joint, and travels toward the posterior surface of the upper portion of the parotid gland, where it may be vulnerable to injury during parotidectomy.

290
Q

Relation of the auriculotemporal nerve to the zygomatic arch

A

It ascends with the superficial temporal vessels over the posterior zygomatic arch and divides into three branches, which provide cutaneous sensory innervation to the tragus, external acoustic meatus, tympanic membrane, and temporal region.

291
Q

The concha and antihelix receive sensory innervation from:

A

The auricular branch of the vagus (X) nerve.

292
Q

Sensory innervation to the helix and lobule is supplied by:

A

Sensory innervation to the helix and lobule is supplied by the great auricular nerve and lesser occipital nerve, which are derived from C2-3

293
Q

The auricularis anterior, superior, and posterior muscles receive motor innervation from:

A

The auricularis anterior, superior, and posterior muscles receive motor innervation from the temporal and posterior auricular branchesof the facial (V) nerve

294
Q

The temporalis muscle is innervated by:

A

The temporalis muscle is innervated by the deep temporal nerves, which are derived from the anterior, or motor, branch of the mandibular division of the trigeminal nerve (V3).

295
Q

Which of the following structures passes though the foramen ovale?
(A) Accessory nerves
(B) Optic (II) nerve
(C) Ophthalmic division of the trigeminal nerve (V1)
(D) Maxillary division of the trigeminal nerve (V2)
(E) Mandibular division of the trigeminal nerve (V3)

A

(E) Mandibular division of the trigeminal nerve (V3)

The mandibular division of the trigeminal nerve (V3) passes through the foramen ovale. This foramen is located in the region of the superior orbital fissure, which contains the nerves to the extraocular muscles, sympathetic fibers, and vessels and is found within the middle cranial fossa.

296
Q
The external auditory meatus develops from which of the following embryologic structures?
(A) First branchial arch
(B) Second branchial arch
(C) Third branchial arch
(D) First branchial groove
(E) Second branchial groove
A

(D) First branchial groove

Development of the six branchial arches occurs within the walls of the anterior foregut during the fourth week of gestation, as neural crest cells migrate into the future head and neck region and alternating ridges and depressions develop. Each branchial arch is composed of endoderm, ectoderm, and mesoderm. During development, a series of clefts forms to create the branchial grooves externally and the pharyngeal pouches internally. The branchial grooves are lined with surface ectoderm and the pharyngeal pouches are lined with foregut endoderm

297
Q

Each branchial arch is composed of:

A

Each branchial arch is composed of endoderm, ectoderm, and mesoderm.

298
Q

Branchial grooves vs pharyngeal pouches

A

During development, a series of clefts forms to create the branchial grooves externally and the pharyngeal pouches internally. The branchial grooves are lined with surface ectoderm and the pharyngeal pouches are lined with foregut endoderm.

299
Q

The first branchial arch is called:

A

Mandibular branchial arch

300
Q

The second branchial arch is called:

A

Hyoid branchial arch

301
Q

The auricle arises from:

A

The first branchial arch gives rise to the anterior (first through third) hillocks, and the second branchial arch gives rise to the posterior (fourth through sixth) hillocks. By the end of the eighth week of gestation, the auricle assumes its characteristic shape following differential growth and fusion of the hillocks.

302
Q

The external auditory meatus develops from:

A

The external auditory meatus develops from the dorsal aspect of the first branchial groove, which is a cleft between the first and second branchial arches

303
Q

Embryology - The cervical sinus develops as a result of:

A

The cervical sinus develops as a result of caudal overgrowth of the second branchial arch

304
Q

What happens to the 2nd, 3rd, 4th branchial grooves?

A

The second, third, and fourth branchial grooves are obliterated within the cervical sinus during the later stages of embryologic development.

305
Q

Which of the following cranial nerves provides innervation to the temporalis muscle?
(A) Ophthalmic division of the trigeminal nerve (V1)
(B) Maxillary division of the trigeminal nerve (V2)
(C) Mandibular division of the trigeminal nerve (V3)
(D) Abducens (VI) nerve
(E) Facial (VII) nerve

A

(C) Mandibular division of the trigeminal nerve (V3)

The temporalis muscle receives its innervation primarily from the branches of the mandibular division of the trigeminal nerve (V3), which then exits the skull via the foramen ovale. The motor branches of the buccal, masseteric, and mandibular nerves are derived from V3 and act to innervate the temporalis muscle.

306
Q

Anatomy of the temporalis muscle

A

This muscle is a large, powerful muscle of mastication that originates along the temporal crest of the skull and inserts into the coronoid process of the mandible.

307
Q

The abducens nerve innervates:

A

Lateral rectus motor innervation

308
Q

What innervates the temporalis muscle?

A

The temporalis muscle receives its innervation primarily from the branches of the mandibular division of the trigeminal nerve (V3), which then exits the skull via the foramen ovale. The motor branches of the buccal, masseteric, and mandibular nerves are derived from V3 and act to innervate the temporalis muscle.

309
Q
Which of the following neural structures does NOT pass through the superior orbital fissure? 
(A) Optic (II) nerve
(B) Oculomotor (III) nerve
(C) Trochlear (IV) nerve
(D) Abducens (VI) nerve
(E) Sympathetic nerve fibers
A

(A) Optic (II) nerve

The superior orbital fissure transmits the oculomotor (III), trochlear (IV), and abducens (VI) nerves and sympathetic nerve fibers from the cavernous plexus. In patients who sustain high-velocity fractures of the orbital roof, the fractures may extend to involve the structures of the superior orbital fissure, resulting in a condition known as superior orbital fissure syndrome. This syndrome manifests as loss of ocularmotion resulting from paralysis of the motor nerves that pass through the superior orbital fissure, but does not affect vision.

310
Q

Superior orbital fissure syndrome

A

In patients who sustain high-velocity fractures of the orbital roof, the fractures may extend to involve the structures of the superior orbital fissure, resulting in a condition known as superior orbital fissure syndrome. This syndrome manifests as loss of ocularmotion resulting from paralysis of the motor nerves that pass through the superior orbital fissure, but does not affect vision.

311
Q

Orbital apex syndrome

A

Orbital apex syndrome, which involves injury to the optic nerve resulting from extension of the fracture into the optic canal, is characterized by loss of vision.

312
Q

What separates the superior orbital fissure and the optic foramen?

A

The lesser wing of the sphenoid bone

313
Q
Which of the following nerves passes through the stylomastoid foramen? 
(A) Facial (VII)
(B) Glossopharyngeal (IX)
(C) Great auricular
(D) Mandibular
(E) Posterior auricular
A

(A) Facial (VII)

The stylomastoid foramen is located anterior to the mastoid process and posterior to the styloid root. The facial nerve and stylomastoid artery pass through the stylomastoid foramen as they exit the temporal bone; from this point, the facial (VII) nerve gives off branches to sites throughout the face. The main trunk of the facial nerve supplies innervation to the posterior belly of the digastric muscle as it leaves the foramen and enters the parotid gland.

314
Q

The posterior auricular nerve general anatomy

A

The posterior auricular nerve branches off from the facial nerve just after it passes through the stylomastoid foramen and supplies innervation to the skin behind the ear.

315
Q

What does the facial nerve supply just after it leaves the _________ foramen?

A

The main trunk of the facial nerve supplies innervation to the posterior belly of the digastric muscle as it leaves the stylomastoid foramen and enters the parotid gland

316
Q
Anesthesia of the nasal side wall is best accomplished through infiltration of which of the following nerves?
(A) Buccal
(B) Dorsal nasa
(C) Infraorbital
(D) Infratrochlear
(E) Zygomaticofacial
A

(C) Infraorbital

The infraorbital nerve, which is the terminal branch of the maxillary division of the trigeminal nerve (V2), supplies sensation to the nasal side wall, ala, upper lip, lower eyelid, and medial aspect of the upper cheek. Anesthesia of this nerve is best accomplished by introducing the needle just lateral to the nasal ala and directing it to a point 0.5 cm below the central section of the infraorbital rim, directly into the foramen rotundum

317
Q

Anesthesia of the infraorbital nerve from the cheek

A

The infraorbital nerve, which is the terminal branch of the maxillary division of the trigeminal nerve (V2), supplies sensation to the nasal side wall, ala, upper lip, lower eyelid, and medial aspect of the upper cheek. Anesthesia of this nerve is best accomplished by introducing the needle just lateral to the nasal ala and directing it to a point 0.5 cm below the central section of the infraorbital rim, directly into the foramen rotundum.

318
Q

Anesthesia of the buccal nerve

A

The buccal nerve is a branch of the mandibular division of the trigeminal nerve (V3). It arises from the surface of the buccinator muscle to supply sensation to the central cheek. Appropriate anesthesia is achieved by injecting the nerve at its origin on the mandibular nerve between the sigmoid notch of the mandible and lateral pterygoid plate.

319
Q

The infratrochlear nerve supplies sensation to:

A

The infratrochlear nerve supplies sensation to the skin of the root of the nose and medial upper eyelid.

320
Q

The dorsal nasal nerve supplies sensation to:

A

The dorsal nasal nerve branches from the anterior ethmoidal nerve, then emerges at the distal end of the nasal bones to supply sensation to the skin of the dorsal nose from that point distally to the nasal tip.

321
Q

General anatomy of the zygomaticofacial nerve

A

The zygomaticofacial nerve is a branch of the zygomatic nerve, which is derived from the maxillary division of the trigeminal nerve (V2). It emerges through one or two foramina just lateral to the infraorbital rim and supplies sensation to the skin over the zygoma and upper portion of the central cheek

322
Q
Which of the following muscles acts to elevate the mandible?
(A) Anterior digastric
(B) Buccinator
(C) Lateral pterygoid
(D) Medial pterygoid
(E) Posterior digastric
A

(D) Medial pterygoid

Contraction of the medial pterygoid muscle elevates the mandible and pulls it medially. This muscle arises inferiorly, laterally, and posteriorly from the medial surface of the pterygoid plate and inserts on the medial ramus and mandibular angle. A patient who sustains a fracture medial to the mandibular angle has displacement of the mandibular ramus medially and cephalad because of the forces of this muscle

323
Q

Actions / insertions of the medial pterygoid muscle

A

Contraction of the medial pterygoid muscle elevates the mandible and pulls it medially. This muscle arises inferiorly, laterally, and posteriorly from the medial surface of the pterygoid plate and inserts on the medial ramus and mandibular angle. A patient who sustains a fracture medial to the mandibular angle has displacement of the mandibular ramus medially and cephalad because of the forces of this muscle.

324
Q

Anatomy of the digastric muscle

A

The anterior belly of the digastric muscle originates from the inside lower border of the symphysis and attaches to the lateral corner of the hyoid bone, while the posterior belly of the digastric extends between the hyoid bone and mastoid notch of the temporal bone.

325
Q

The digastric muscle in a mandibular fracture

A

The digastric muscle is part of the suprahyoid musculature, which in a patient with a mandibular fracture would pull the anterior mandibular fragments posteroinferiorly.

326
Q

Origin/insertion of the buccinator muscle

A

The buccinator muscle arises from the pterygomandibular raphae to insert into the orbicularis oris muscle and the mucosa and skin of the lips.

327
Q

Action of the buccinator muscle

A

This muscle of facial expression is innervated by the buccal branches of the facial nerve and acts to flatten the cheek against the teeth.

328
Q

Origin/insertion of the lateral pterygoid muscle

A

The lateral pterygoid muscle arises from the lateral pterygoid plate of the sphenoid bone and extends posteriorly and horizontally to insert on the neck of the condyle and into the anterior margin of the articular disk of the temporomandibular joint.

329
Q

Actions of the lateral pterygoid muscle

A

As the condyle is moved forward, the lateral pterygoid acts to protrude the mandible, then slides down the articular eminence, opening the mouth.

330
Q

The lateral pterygoid in a subcondylar fracture

A

In patients with subcondylar fractures, the muscle pulls the condylar head in an anteromedial direction

331
Q

The lateral pterygoid in a unilateral mandibular fracture

A

In patients with unilateral mandibular fractures the contralateral pterygoid continues to protrude, resulting in deviation of the mandible to the side of the fracture.

332
Q
Which of the following branchial arches gives rise to the stylopharyngeus muscle?
(A) First
(B) Second
(C) Third
(D) Fourth
(E) Fifth
A

(C) Third

The stylopharyngeus is the only muscle derived from the third branchial arch. This muscle, which receives its innervation from the glossopharyngeal (X) nerve, is part of the intrinsic musculature of the larynx

333
Q

What is the only muscle derived from the third branchial arch?

A

The stylopharyngeus

334
Q

The stylopharyngeal muscle is innervated by the:

A

Glossopharyngeal nerve

335
Q

Muscles from the first branchial arch are innervated by:

A

Trigeminal nerve (V)

336
Q

Muscles derived from the first branchial arch

A

The muscles of mastication, the anterior belly of the digastric, the mylohyoid, the tensor tympani, and the tensor veli palatini are derived from the first branchial arch

337
Q

Muscles from the second branchial arch are innervated by:

A

Facial nerve (VII)

338
Q

Muscles derived from the second branchial arch

A

The muscles of facial expression, the posterior belly of the digastric, the stylohyoid, and the stapedius are derived from the second branchial arch

339
Q

Muscles from the fourth branchial arch are innervated by:

A

Vagus nerve (X)

340
Q

Muscles derived from the fourth branchial arch

A

The fourth branchial arch gives rise to the constrictor muscles of the pharynx, the cricothyroid, the levator veli palatini, the palatopharyngeus, and the palatoglossus.

341
Q

The fifth branchial arch

A

The fifth branchial arch is only present in rudimentary forms and is often combined with the sixth branchial arch during any embryologic investigation.

342
Q
The hamulus bone acts as a pulley for which of the following muscles?
(A) Levator veli palatini
(B) Palatoglossus
(C) Palatopharyngeus
(D) Superior pharyngeal constrictor
(E) Tensor veli palatini
A

(E) Tensor veli palatini

The hamulus, which is an inferior extension of the medial pterygoid plate, acts as a pulley for the tensor veli palatini muscle. This muscle arises from the medial pterygoid plate and consolidates into a tendon that changes course as it wraps around the hamulus, and then extends medially to form an aponeurosis in the anterior soft palate. The tensor veli palatini also inserts on the posterior hard palate.

343
Q

Linkage between the levator veli palatini and the tensor veli palatini

A

A fibrous linkage exists between the fibers of the levator veli palatini and the tensor veli palatini; therefore, a patient who sustains a fracture of the pterygoid hamulus may have increased medial displacement of these muscles.

344
Q

The superior pharyngeal constrictor anatomy

A

The superior pharyngeal constrictor muscle arises from the posterior aspect of the medialpterygoid plate and from the hamulus, pterygomandibular raphe, and alveolar process of the mandible. The fibers of this muscle wrap around the pharynx and insert on the median raphe to form Passavant’s ridge, which contributes to velopharyngeal closure.

345
Q

Passavant’s ridge contributes to:

A

Passavant’s ridge contributes to velopharyngeal closure.

346
Q
The internal carotid artery supplies which of the following arteries of the scalp?
(A) Frontalis
(B) Occipitalis
(C) Parietal
(D) Posterior auricular
(E) Supraorbital
A

(E) Supraorbital

The supraorbital and supratrochlear arteries are branches of the internal carotid artery via the ophthalmic artery and, therefore, receive their blood supply from the internal carotid.

347
Q

Branches of the internal carotid artery to the scalp

A

The supraorbital and supratrochlear arteries are branches of the internal carotid artery via the ophthalmic artery and, therefore, receive their blood supply from the internal carotid.

348
Q

The arteries of the scalp travel through the _______________ in what direction?
Eventual anastomosis?

A

The arteries of the scalp travel through the subcutaneous fat from the periphery toward the vertex, then anastomose in the midline with branches of the ophthalmic artery.

349
Q

The arteries of the scalp anastomose with what?

A

Branches of the ophthalmic artery

350
Q

If the internal carotid artery is thrombosed, what supplies the anterior scalp?

A

If the internal carotid artery is thrombosed, branches of the external carotid arteries supply blood to the anterior part of the scalp through the angular and ophthalmic arteries.

351
Q

The frontalis artery branches from:

A

The frontal is a branch of the superficial temporal artery, which in turn branches from the external carotid arteries.

352
Q

The parietal artery branches from:

A

The parietal is a branch of the superficial temporal artery, which in turn branches from the external carotid arteries.

353
Q
The levator palpebrae superioris muscle originates from which of the following structures?
(A) Annulus tendineus
(B) Bulbar fascia
(C) Ethmoid bone
(D) Frontal bone
(E) Sphenoid bone
A

(E) Sphenoid bone

The seven ocularmuscles in the orbit are the levator palpebrae superioris, the superior and inferior oblique, and the superior, inferior, medial, and lateral recti. The rectus muscles and the superior oblique arise from the fibrous ring around the optic nerve (the annulus tendineus communis). In contrast, the inferior oblique arises from the maxilla, while the levator palpebrae superioris arises from the lesser wing of the sphenoid bone.

354
Q

What forms the annulus tendineus communis?

A

The rectus muscles and the superior oblique arise from the fibrous ring around the optic nerve (the annulus tendineus communis)

355
Q

Which rectus muscle does not participate in the annulus tendineus communis?

A

The inferior oblique - it arises from the maxilla

356
Q

Origin of the levator palpebrae superioris

A

The levator palpebrae superioris arises from the lesser wing of the sphenoid bone.

357
Q
Which of the following structures is derived from the second branchial arch?
(A) Incus
(B) Malleus
(C) Sphenoid (greater wing)
(D) Stapes
(E) Temporal bone (squamous portion)
A

(D) Stapes

The stapes is derived from Reichert’s cartilage, which is the precursor of the second branchial arch. Other structures derived from the second branchial arch include the stylohyoid process, the lesser cornu of the hyoid bone, and the upper part of the body of the hyoid bone. The other two ossicles of the middle ear (malleus and incus) are derived from the first branchial arch. Cartilage precursors in the firstbranchial arch are the quadrate cartilage and Meckel’s cartilage, both of which give rise to skeletal elements through endochondral ossification. The quadrate cartilage gives rise to the greater wing of the sphenoid bone and the incus, while Meckel’s cartilage gives rise to the malleus and the mandibular condyles.

358
Q

The stapes is derived from:

A

The stapes is derived from Reichert’s cartilage, which is the precursor of the second branchial arch.

359
Q

Structures derived from the second branchial arch:

A

Structures derived from the second branchial arch include the stapes, the stylohyoid process, the lesser cornu of the hyoid bone, and the upper part of the body of the hyoid bone.

360
Q

Origin of the ossicles of the middle ear:

A

Malleus and incus - 1st branchial arch

Stapes - 2nd branchial arch

361
Q

Cartilage precursors in the firstbranchial arch are:

A

Cartilage precursors in the firstbranchial arch are the quadrate cartilage and Meckel’s cartilage, both of which give rise to skeletal elements through endochondral ossification.

362
Q

The skeletal elements of the maxillary and mandibular prominences originate:

A

The skeletal elements of the maxillary and mandibular prominences originate from intramembranous ossification, which is derived from direct ossification of the dermal mesenchyme of the arches.

363
Q

The maxillary prominence gives rise to:

A

The maxillary prominence gives rise to the maxilla, zygoma, and squamous portion of the temporal bone.

364
Q

The mandibular prominence gives rise to:

A

The mandibular prominence gives rise to the body and ramus of the mandible

365
Q
Which of the following structures drains into the inferior meatus?
(A) Anterior ethmoid air cells
(B) Frontal sinus
(C) Maxillary sinus
(D) Nasolacrimal duct
(E) Sphenoid sinus
A

(D) Nasolacrimal duct

The nasolacrimal duct drains into the inferior meatus. In contrast, the anterior ethmoid air cells and maxillary sinus drain into the middle meatus; the frontal sinus also drains into the middle meatus via the frontonasal duct. The sphenoid sinus drains into the sphenoethmoid recess, which is found above and behind the superior concha.

366
Q
A patient sustains a laceration of the superior helix of the auricle. Adequate anesthesia is most likely to be obtained with block of which of the following nerves?
(A) Facial (VII)
(B) Glossopharyngeal (IX)
(C) Great auricular
(D) Trigeminal (V)
(E) Vagus (X)
A

(D) Trigeminal (V)

Adequate anesthesia is most likely to be obtained with a block of the auriculotemporal nerve, which is a branch of the trigeminal (V) nerve that supplies sensation to the superior aspect of the helix.

The auriculotemporal nerve, as well as C2 and C3, supply sensation to the outer portions of the ear.

367
Q
Which of the following anatomic structures of the ear originates from the second (hyoid) pharyngeal arch?
(A) Antitragus
(B) Helical root
(C) Superior helix
(D) Tragus
A

(A) Antitragus

During the sixth week of fetal gestation, the anatomic subunits of the auricle arise from six hillocks that are derived from the first and second pharyngeal arches. The anterior three hillocks from the first (or mandibular) pharyngeal arch ultimately develop into the tragus, helical root, and superior helix. The antihelix, antitragus, inferior helix, and lobule form from the fourth through sixth posterior hillocks from the second (or hyoid) pharyngeal arch.

368
Q

When is the ear formed?

A

During the sixth week of fetal gestation, the anatomic subunits of the auricle arise from six hillocks that are derived from the first and second pharyngeal arches.

369
Q

Parts of the ear formed from the anterior three hillocks

A

The anterior three hillocks from the first (or mandibular) pharyngeal arch ultimately develop into the tragus, helical root, and superior helix.

370
Q

Parts of the ear formed from the posterior three hillocks

A

The antihelix, antitragus, inferior helix, and lobule form from the fourth through sixth posterior hillocks from the second (or hyoid) pharyngeal arch.

371
Q

The structures that arise from the first pharyngeal arch typically drain into:

A

The structures that arise from the first pharyngeal arch typically drain into the parotid lymph nodes

372
Q

The structures from the second pharyngeal arch drain into:

A

The structures from the second pharyngeal arch drain into the cervical lymph nodes.

373
Q
The palatine bone contributes to which of the following structures?
(A) Anterior nasal spine
(B) Dental arch
(C) External nose
(D) Hamulus
(E) Orbit
A

(E) Orbit

The palatine bone is located posterior to the maxilla and pterygoid lamina; it is composed of horizontal and pyramidal processes. The horizontal process contributes to the posterior aspect of the hard palate and becomes the floor of the choana. The pyramidal process extends vertically to contribute to the floor of the orbit.

374
Q

Horizontal process of the palatine bone:

A

The horizontal process contributes to the posterior aspect of the hard palate and becomes the floor of the choana.

375
Q

Pyramidal process of the palatine bone:

A

The pyramidal process extends vertically to contribute to the floor of the orbit

376
Q

The anterior nasal spine is a prominence of:

A

The anterior nasal spine is a prominence of the maxillary bone

377
Q

The external nose is comprised of which bones?

A

The external nose is comprised of the paired nasal bones and frontal processes of the maxilla.

378
Q
An 8-year-old boy has had a painless mass in the midline of the neck that has slowly enlarged over the past two years. This finding is most consistent with
(A) branchial cleft cyst
(B) dermoid cyst
(C) laryngocele
(D) lymphatic malformation
(E) thyroglossal duct cyst
A

(E) thyroglossal duct cyst

This 8-year-old boy has a thyroglossal duct cyst, a slowly enlarging, painless mass of the midline of the neck that results from incomplete degeneration of the thyroglossal duct before birth. This duct connects an endodermal diverticulum (which later becomes the thyroid gland) to the foramen cecum in the developing fetus; in most cases, the duct degenerates once the diverticulum becomes a solid gland.

379
Q

What does the thyroglossal duct connect / anatomy

A

This duct connects an endodermal diverticulum (which later becomes the thyroid gland) to the foramen cecum in the developing fetus

380
Q

Most thyroglossal duct cysts are located:

A

Although thyroglossal duct cysts can be found from the base of the tongue to the cricoid cartilage, most cysts are located inferior to the hyoid bone.

381
Q

Definitive treatment of a thyroglossal duct cyst

A

Definitive management of a thyroglossal duct cyst is complete surgical excision of the cyst and central portion of the hyoid bone; this is known as the Sistrunk procedure

382
Q

Sistrunk procedure

A

Definitive management of a thyroglossal duct cyst is complete surgical excision of the cyst and central portion of the hyoid bone; this is known as the Sistrunk procedure

383
Q

Laryngocele

A

A laryngocele is an air pocket that normally communicates with the larynx and often enlarges with coughing or other changes in pressure within the trachea

384
Q
Patients with paralysis of the trigeminal nerve have loss of function of which of the following muscles?
(A) Levator veli palatini
(B) Palatopharyngeus
(C) Stylohyoid
(D) Stylopharyngeus
(E) Tensor veli palatini
A

(E) Tensor veli palatini

Because the tensor veli palatini muscle receives its motor innervation from the trigeminal (V) nerve, a patient who has paralysis of this nerve will experience a loss of function of the tensor veli palatini, which is a derivative of the first branchial arch.

In contrast, the anatomically adjacent levator veli palatini muscle is a derivative of the fourth branchial arch and is innervated bythe superior laryngeal branch of the vagus (X) nerve. The palatopharyngeus muscle has this same origin and innervation. The stylohyoid muscle is derived from the second branchial arch and innervated by the facial (VII) nerve, while the stylopharyngeus muscle is derived from the third branchial arch and innervated by the glossopharyngeal (IX) nerve.

385
Q

A 55-year-old woman has numbness of the earlobe after undergoing biopsy of an internal jugular lymph node. The most likely cause is injury to which of the following structures?
(A) Auricular branch of the vagus nerve
(B) Auriculotemporal nerve
(C) Great auricular nerve
(D) Posterior auricular nerve
(E) Ventral ramus of the first cervical root

A

(C) Great auricular nerve

This 55-year-old woman has numbness of the earlobe due to injury to the great auricular nerve. This nerve arises from the second and third cervical nerves and emerges from the posterior border of the sternocleidomastoid muscle, then travels anterosuperiorly between the sternocleidomastoid and platysma muscles and divides into auricular, facial, and mastoid branches. The auricular branch provides sensation to the earlobe and posterior two thirds of the ear and is prone to injury during surgery in the region of the upper lateral neck.

386
Q

Course of the great auricular nerve

A

The great auricular nerve arises from the second and third cervical nerves and emerges from the posterior border of the sternocleidomastoid muscle, then travels anterosuperiorly between the sternocleidomastoid and platysma muscles and divides into auricular, facial, and mastoid branches. The auricular branch provides sensation to the earlobe and posterior two thirds of the ear and is prone to injury during surgery in the region of the upper lateral neck.

387
Q

The auricular branch of the vagus nerve arises from the _________ and receives a contribution from the ___________

A

The auricular branch of the vagus nerve, also known as Arnold’s nerve, arises from the superior ganglion, receives a contribution from the glossopharyngeal nerve

388
Q
Which of the following structures is a branch of the mandibular division of the trigeminal nerve?
(A) Infraorbital nerve
(B) Lingual nerve
(C) Nasopalatine nerve
(D) Posterosuperior alveolar nerve
(E) Posterosuperior nasal nerve
A

(B) Lingual nerve

The lingual nerve, which supplies sensation to the anterior two thirds of the tongue, is a branch of the mandibular division of the trigeminal nerve (V3).

389
Q

Structures that arise from CN V3 and their innervation

A

The lingual nerve: sensation to the anterior two thirds of the tongue

The inferior alveolar nerve: sensation to the mandibular teeth

The long buccal branch: sensation to the buccal mucosa

The mental nerve: sensation to the skin of the chin and lower lip and the mucosa of the lip and adjacent gingiva.

Auriculotemporal nerve: Sensory to the anterior auricle, a large portion of the temporal region, and part of the external auditory canal and gives off a branch to supply the TMJ.

390
Q
A 42-year-old man is scheduled to undergo surgical excision of a lesion of the lower lip. During anesthetic blockade of the mental nerve prior to the procedure, the nerve foramen can be located beneath the apex of which of the following mandibular teeth?
(A) Central incisor
(B) Cuspid
(C) First molar
(D) Lateral incisor
(E) Second bicuspid
A

(E) Second bicuspid

During anesthetic blockade of the mental nerve, the nerve foramen can be found beneath the apex of the second bicuspid tooth. This nerve, which is the terminal branch of the inferior alveolar nerve, supplies sensory innervation to the skin and mucous membranes of the lower lip as well as the skin of the anterior mandible and chin.

391
Q

During anesthetic blockade of the mental nerve, the nerve foramen can be found beneath the apex of:

A

During anesthetic blockade of the mental nerve, the nerve foramen can be found beneath the apex of the second bicuspid tooth. This nerve, which is the terminal branch of the inferior alveolar nerve, supplies sensory innervation to the skin and mucous membranes of the lower lip as well as the skin of the anterior mandible and chin.

392
Q

Stensen’s duct can be found at which of the following anatomic sites?
(A) At the mandibular angle
(B) At the pre auricular border
(C) At the zygomatic arch
(D) Between the superficial and deep lobes of the parotid gland
(E) Within the buccal space

A

(E) Within the buccal space

Stensen’s duct can be found within the buccal space, which is bordered anteriorly by the orbicularis oris muscle, posteriorly by the edge of the masseter muscle, superiorly by the zygomaticus major muscle, and inferiorly by the fascial attachment of the buccinator muscle to the mandible.

The duct develops deep within the parotid gland and emerges from the superior third of the gland at its anterior border, then courses below the zygomatic arch and enters the buccal space, inserting into the buccinator and then entering the oral cavity opposite the upper second molar.

The facial artery and vein, buccal branches of the facial nerve, and buccal fat pad can also be found within the buccal space.

393
Q

The buccal space is bordered by:

A

The buccal space is bordered anteriorly by the orbicularis oris muscle, posteriorly by the edge of the masseter muscle, superiorly by the zygomaticus major muscle, and inferiorly by the fascial attachment of the buccinator muscle to the mandible.

394
Q

The parotid duct course:

A

The duct develops deep within the parotid gland and emerges from the superior third of the gland at its anterior border, then courses below the zygomatic arch and enters the buccal space, inserting into the buccinator and then entering the oral cavity opposite the upper second molar.

395
Q
Each of the following is a muscle of mastication EXCEPT the
(A) buccinator
(B) lateral pterygoid
(C) masseter
(D) medial pterygoid
(E) temporalis
A

(A) buccinator

The lateral and medial pterygoids, masseter, and temporalis muscles are muscles of mastication, capable of exerting force on the mandible. These muscles are innervated by the mandibular division of the trigeminal nerve (V3).

The buccinator muscle arises from the alveolar process of the maxilla, from the mandible opposite the molars, and from the anterior border of the pterygoid mandibular raphe; it inserts into the submucosa of the cheeks and lips and is used to compress the cheeks against the teeth and gums. It is innervated by the facial (VII) nerve. Although it is not a muscle of mastication, it assists the tongue in directing food between the molars during mastication.

396
Q

Muscles of mastication

A

The lateral and medial pterygoids, masseter, and temporalis muscles aremuscles of mastication, capable of exerting force on the mandible. These muscles are innervated by the mandibular division of the trigeminal nerve (V3).

397
Q

Masseter muscle anatomy

A

The masseter muscle arises from the zygomatic arch and inserts on the mandibular ramus and coronoid process; during mastication it elevates the mandible to occlude the teeth

398
Q

Buccinator role in mastication

A

Although it is not a muscle of mastication, it assists the tongue in directing food between the molars during mastication

399
Q

Buccinator muscle anatomy

A

The buccinator muscle arises from the alveolar process of the maxilla, from the mandible opposite the molars, and from the anterior border of the pterygoid mandibular raphe; it inserts into the submucosa of the cheeks and lips and is used to compress the cheeks against the teeth and gums.

400
Q
Which of the following provides the drainage ostium for the nasolacrimal duct into the nose?
(A) Inferior meatus
(B) Middle meatus
(C) Superior meatus
(D) Supreme meatus
A

(A) Inferior meatus

The inferior meatus provides the drainage ostium for the nasolacrimal duct into the nose. The maxillary, frontal, and anterior ethmoidal sinuses drain into the middle meatus, while the posterior ethmoid cells drain into the superior meatus. The sphenoid sinus drains into the supreme meatus, also referred to as the sphenoethmoidal recess.

401
Q
A 60-year-old man undergoes microvascular anastomosis. The proximal facial artery off the external carotid artery is to be dissected and used as a recipient vessel. During the procedure, a large, overlying, nerve-like structure is inadvertently transected. Which of the following is the most likely consequence?
A) Dysarthria
B) Lip elevation
C) Shoulder drop
D) Tongue numbness
E) Vocal cord paralysis
A

A) Dysarthria

The facial artery generally starts as part of the lingual-facial trunk, then travels below the hypoglossal nerve before it enters into the submandibular gland and along the lateral border of the mandible. Failure to recognize this structure could cause injury and subsequent loss of motor function of the ipsilateral tongue. Ipsilateral hypoglossal (XII) nerve injury causes the tongue to move toward the side of damage, resulting in dysarthria, and problems moving solid food to the oropharynx.

Vocal cord paralysis is related to a recurrent laryngeal or vagus (X) nerve injury, which could happen after superior laryngeal artery or common carotid dissection, respectively.

Shoulder drop is related to accessory (XI) nerve injury, which has anatomic relation to the occipital artery.

Tongue numbness is from an injury to the lingual nerve (related mostly to the laryngeal artery and submandibular duct).

Lip elevation is related to a marginal mandibular (V3) nerve injury—this nerve runs with the facial artery lateral to the mandible, but not below the margin of the mandible.

402
Q
An 11-month-old male infant is noted by his parents to have a painless, progressive, right maxillary growth. At an outside facility, an incisional biopsy is performed. The pathology shows sinonasal myxoma. Postoperative MRI shows residual tumor with surrounding inflammation. Which of the following is the most appropriate treatment plan?
A) Chemotherapy and radiation therapy
B) Chemotherapy only
C) Curettage debulking of the tumor
D) Surgical resection with clear margin
E) Observation
A

D) Surgical resection with clear margin

Myxomas are slow-growing benign tumors. When they present in the infant face, they are most common in the maxilla or mandible. They present as a painless, progressive facial swelling and should be surgically removed with a clear margin. These tumors should have a clear margin to prevent incomplete resection and continued growth. They are not always well circumscribed, so a normal margin or tissue plane should be resected with the tumor.

403
Q

A 20-year-old woman with a history of bruxism is evaluated because of a 3-year history of gradual widening of the lower third of the face. Physical examination shows rectangular appearance of the face; occlusion shows no abnormalities. Anteroposterior x-ray study discloses bone spurs at both angles of the mandible. Which of the following is the most appropriate next step in management?
A) Excision of the submandibular gland
B) Injection of botulinum toxin type A to the masseter muscle
C) Marginal mandibulectomy
D) Suction-assisted lipectomy of the cheek
E) Superficial parotidectomy

A

B) Injection of botulinum toxin type A to the masseter muscle

The patient described has bilateral masseter hypertrophy. Treatment options for this condition include muscle relaxants, injection of botulinum toxin type A, or resection of the internal layer of the masseter muscle.

Superficial parotidectomy is indicated for benign and malignant tumors of the parotid gland.

Resection of the submandibular gland is indicated for recurrent sialadenitis (infection) or obstructive sialodocholithiasis (salivary stones), as well as for benign tumors such as pleomorphic adenomas.

Marginal mandibulectomy may be indicated for certain benign and malignant tumors of the intraoral cavity.

Suction-assisted lipectomy will not treat masseter hypertrophy.

404
Q
A 22-year-old man is brought to the emergency department after sustaining a stab wound to the face. The patient is hemodynamically stable, and physical examination shows a laceration that extends from the tragus of the right ear to the right oral commissure. Which of the following is the most likely primary complication of saliva extravasating into the wound because of parotid duct injury?
A) Parotid gland atrophy
B) Salivary fistula
C) Sialocele
D) Wound infection
E) Xerostomia
A

C) Sialocele

If parotid duct injury is not repaired immediately, saliva can leak into the surrounding soft tissues. This leakage most commonly increases the risk for sialocele (pseudocapsule), followed by salivary fistula formation. Wound infection, parotid gland atrophy, and xerostomia (dry mouth) are uncommon. Studies have shown that correction of the more common complications may require surgical or medical treatments such as use of anti-sialogogues, radiation therapy, parasympathetic denervation (tympanic denervation), cauterization of the fistulous tract, reconstruction of the duct, or superficial or total parotidectomy.

405
Q
An otherwise healthy term 6-month-old male infant is evaluated for a mobile, firm, well-circumscribed mass at the right lateral brow in the area of the zygomaticofrontal suture. Which of the following procedures is the most appropriate next step in management?
A) CT scan
B) Fine-needle aspiration of the mass
C) MRI
D) Surgical excision of the mass
E) Ultrasonography
A

D) Surgical excision of the mass

Dermoid cysts are common in the lateral brow. They present as firm, well-circumscribed, slow-growing masses that have the potential for infection or continued growth. Surgical excision is recommended and no imaging is required. CT scan or MRI require sedation and are unnecessary risks for this patient with a lateral dermoid. Midline masses do require imaging because of the risk for intracranial excision.

406
Q
A 24-year-old man with a history of left facial trauma and condylar fracture of the mandible is evaluated because of redness and perspiration of the left cheek and ear after ingesting certain foods. Aberrant regeneration of which of the following nerves is the most likely cause of this patient’s symptoms?
A) Auriculotemporal
B) Facial
C) Great auricular
D) Inferior alveolar
E) Lingual
A

A) Auriculotemporal

Although it is most commonly seen after parotidectomy, Frey syndrome has also been encountered after condylar fracture of the mandible and treatment. The syndrome is thought to result from damage to auriculotemporal parasympathetic nerve fibers with subsequent aberrant regeneration and innervation of sympathetic fibers to the sweat glands.

The facial, inferior alveolar, greater auricular, and lingual nerves are not thought to be the underlying cause of Frey syndrome.

407
Q
A 12-hour-old male newborn has cyanosis that improves with crying. Which of the following is the most likely diagnosis?
A) Choanal atresia
B) Laryngomalacia
C) Macroglossia
D) Micrognathia
E) Subglottic stenosis
A

A) Choanal atresia

Choanal atresia is a unilateral or bilateral anatomic abnormality of the posterior nasal passages and choanae, which prevents nasal gas exchange in newborns. The classic presentation of bilateral choanal atresia is cyanosis that improves with crying. Nasal airway obstruction can also become apparent when attempting to breast-feed the baby. On clinical examination, there would be no fogging of a mirror when held under the nares. The remaining abnormalities are other causes of respiratory obstruction in the pediatric patient.

408
Q

A 54-year-old man comes to the office because of swelling of the left side of the face 3 days after cholecystectomy. Physical examination shows erythema and purulent drainage from the parotid duct. Which of the following is the most appropriate initial management?
A) Antibiotic therapy and sialogogues
B) Aspiration of the mass
C) Incision and drainage of the mass
D) Oral cultures and oral cavity antibiotic irrigation
E) Superficial parotidectomy

A

A) Antibiotic therapy and sialogogues

Initial treatment of acute suppurative sialadenitis begins with aggressive medical management. This includes prompt fluid and electrolyte replacement, oral hygiene, reversal of salivary stasis, and antimicrobial therapy. Stimulation of salivary flow is done by use of sialogogues such as lemon drops. Warm soaks and massage promote secretion and drainage of the gland.

Oral cultures are typically contaminated by oral flora and therefore do not direct antibiotic treatment. Needle aspiration is more accurate in isolating the cause of suppurative parotitis.

Incision and drainage is reserved for cases resistant to medical management. Surgical removal of the gland is not recommended in the case of an actively infected gland.

409
Q
A 65-year-old man undergoes surgery for management of a subtotal massive squamous cell carcinoma of the posterior larynx. History includes chemoradiation and subsequent bilateral selective neck dissections for persistent disease 9 months ago. An anterolateral thigh free flap is chosen for reconstruction, but the operative notes state that both external carotid systems were sacrificed. Which of the following recipient vessels is most appropriate in this patient?
A) Facial
B) Internal mammary
C) Occipital
D) Subclavian
E) Transverse cervical
A

E) Transverse cervical

When performing head and neck microsurgery, a strong background in the vascular anatomy of that region is imperative. As chemotherapy regimens have become commonplace for laryngopharyngeal cancers, so have the challenges of failures which generally require surgery. These cases have much higher complication rates, including fistulas, strictures, and carotid injury, among others. This case demonstrates another complexity that is increasingly observed, the “vascular or vessel-depleted neck.” The facial and occipital arteries are branches of the external system and would not be available. The subclavian is generally not a viable option due to its size, location, and potential complications through dissection. The internal mammary system has potential, but requires dissection through the ribs and has morbidities and the potential need of vein grafts. Generally speaking, even radical neck dissections do not sacrifice the transverse cervical vessels as they are usually used as the caudal margin. A number of reports have detailed the usefulness of these vessels as recipients in cases like the one described.

410
Q
Which of the following cranial nerves provides parasympathetic innervention of the parotid gland?
A) V
B) VII
C) VIII
D) IX
E) X
A

D) IX

Innervation of the parotid gland comes from parasympathetic fibers that travel with the glossopharyngeal nerve (cranial nerve IX). It also receives taste sensation (afferent) from the posterior one-third of the tongue.

The maxillary nerve of cranial nerve V (V2) is a sensory nerve and receives sensation from the mid face.

Parasympathetic fibers (efferent) innervate the submandibular and sublingual glands via the chorda tympani. Afferent fibers, via the chorda tympani, send taste sensation of the anterior two-thirds of the tongue.

The vestibulocochlear nerve (cranial nerve VIII) supplies sound and equilibrium to the brain.

The auricular branch of the vagus nerve (cranial nerve X), innervates the external acoustic meatus. Stimulation of the vagus nerve can lead to reflex coughing (Arnold reflex).

411
Q
Which of the following muscles is associated with the hyoid or second branchial arch?
A) Lateral pterygoid
B) Levator veli palatini
C) Posterior digastric
D) Stylopharyngeus
E) Thyroarytenoid
A

C) Posterior digastric

The first branchial arch, also known as the mandibular arch, has the trigeminal nerve (ophthalmic, maxillary, and mandibular branches) as its neurologic component. The muscles of mastication (i.e., temporalis, masseter, medial, and lateral pterygoids); mylohyoid; anterior digastric; tensor tympanic; and the tensor veli palatini are the muscle components. The cartilaginous bar gives rise to the premaxilla; maxilla, zygomatic bone; part of the temporal bone; incus; malleus; anterior malleolar ligament; and the sphenomandibular ligament. The pharyngeal pouch and groove develop the tubotympanic recess (tympanic cavity, mastoid antrum and pharyngotympanic tube, internal acoustic meatus, tympanic membrane, adenoids). The vascular element largely disappears, but gives rise to the maxillary and external carotid arteries.

The second branchial arch, also known as the hyoid arch, accounts for 95% of all branchial arch anomalies. The cranial nerve is the facial nerve. It supplies the muscles of facial expression; buccinators; stapedius; stylohyoid; posterior digastric; auricular and platysma muscles. The skeletal contributions from Reichert’s cartilage include the stapes, styloid process, stylohyoid ligament, and hyoid (lesser cornu and upper part of body). The pharyngeal pouch and groove shape the crypts of the palatine tonsil and the cervical sinus. The vascular component again primarily disappears but forms the stapedial and hyoid arteries.

Third branchial cleft anomalies are rare. The glossopharyngeal nerve sends motor innervation to only the stylopharyngeus. The cartilaginous bar forms the hyoid (greater cornu and lower part of body). The pharyngeal pouch and groove give rise to the inferior parathyroids, thymus, and cervical sinus. The vascular elements contribute to the internal carotid and common carotid.

The fourth branchial arch is supplied by the vagus nerve (superior laryngeal, inferior laryngeal). Musculature innervated includes the cricothyroid and all intrinsic muscles of the soft palate, including the levator veli palatini. The thyroid and epiglottic cartilage develop from the cartilaginous bar. The pharyngeal pouch and groove form the superior parathyroids, and the thyroid parafollicular cells. The right fourth aortic arch forms the subclavian artery, while the left fourth aortic arch forms the aortic arch.
The sixth branchial arch also is supplied by the vagus nerve (recurrent laryngeal nerve). This area supplies all intrinsic muscles of the larynx (except the ciricothyroid-fourth arch). This includes the thyroarytenoid muscle, which makes up the primary mass of the vocal fold. It consists of two parts, the ventricularis and vocalis. Skeletal derivations form the cricoid, arytenoid, corniculate, and cuneiform cartilages. The right sixth aortic arch gives rise to the right pulmonary artery and the left sixth aortic arch forms the left pulmonary artery and the ductus arteriosus.

412
Q
A 6-year-old boy is brought to the emergency department because of a laceration of the hard palate. Repair of the laceration with local anesthesia for greater palatine nerve block is planned. As the anterior portion is sutured in place, the patient feels pain. Which of the following additional nerve blocks is most appropriate?
A) Anterior superior alveolar
B) Infraorbital
C) Lesser palatine
D) Middle superior alveolar
E) Sphenopalatine
A

E) Sphenopalatine

The sphenopalatine nerve arises from the incisive foramen and provides sensation to the anterior hard palate. Blockade of this nerve is essential for adequate blockade of the palatal mucosa for laceration repair.

The anterior superior alveolar nerve arises from the second branch of the trigeminal nerve before it exits the infraorbital foramen. The nerve supplies the maxillary anterior teeth and is part of the superior dental plexus of nerves that also includes the middle superior alveolar and the posterior superior alveolar nerves.

The infraorbital nerve provides sensation to the ipsilateral lateral nose, upper lip, and cheek.

The lesser palatine descends through the greater palatine foramen and provides innervation to the soft palate and uvula.

413
Q

A 4-year-old boy is brought for evaluation because his mother is concerned about a growth on his neck. Physical examination shows a nontender mass in the midline of the neck that moves vertically when the patient swallows. Which of the following is the most likely cause of this patient’s condition?
A) Failure of vascular apoptosis at 12 weeks’ gestation
B) Failure of the thyroglossal duct to atrophy
C) Ossification of cartilage from the second and third pharyngeal arches
D) Persistent ectopic parathyroid tissue in the neck
E) Persistent ectopic parathyroid tissue in the neck

A

B) Failure of the thyroglossal duct to atrophy

The patient described has a persistent thyroglossal duct cyst.

The thyroid gland is the first of the body’s endocrine glands to develop, at approximately 24 days’ gestation. The gland originates as a proliferation of endodermal epithelial cells on the median surface of the developing pharyngeal floor known as the foramen cecum. The foramen cecum originates from between the first and second pouches and represents the opening of the thyroglossal duct into the tongue. Descent of the thyroid gland carries it anterior to the hyoid bone and anterior to the laryngeal cartilages. As the thyroid gland descends, it forms its mature shape. The thyroid completes its descent in the seventh gestational week, coming to rest in its final location immediately anterior to the trachea.

If the thyroglossal duct does not atrophy, then the remnant can manifest clinically as a thyroglossal duct cyst. While half of these generally midline cystic masses are located at or just below the level of the hyoid bone, they may be located and can track anywhere from the thyroid cartilage up the base of the tongue. Because the hyoid bone develops in an anterior direction and may surround the thyroglossal duct, the surgeon should resect the central portion on the hyoid bone along with the cyst.

The thyroid gland is ensheathed by the visceral fascia, which attaches it firmly to the laryngoskeleton (i.e., Berry ligament). This firm attachment of the gland to the laryngoskeleton is responsible for movement of the thyroid gland and related structures during swallowing. This also causes a thyroglossal duct cyst to move on physical examination.

Ectopic thyroid gland may occur anywhere along the path of initial descent of the thyroid, although it is most common at the base of the tongue, just posterior to the foramen cecum.

Ectopic parathyroid glands occur in 15 to 20% of patients. The glands may be located anywhere near or even within the thyroid or thymus. For example, if parathyroid IVs do not descend entirely, they may be located as high as the bifurcation of the common carotid artery. Conversely, if parathyroid IVs do not release from the thymus, they may be located intrathoracically, as low as the aortopulmonary window. Other common ectopic locations include the anterior mediastinum, posterior mediastinum, and retroesophageal and prevertebral regions. However, even when the parathyroid glands are in an ectopic location, they still often are symmetrical from side to side, making localization somewhat easier.

Ossification of cartilages from the second and third pharyngeal arches gives rise to the hyoid bone.

414
Q
A 32-year-old woman comes to the office for consultation regarding cosmetic improvement of her nose. On examination, facial animation (smiling) causes marked descent of the nasal tip, shortening of the upper lip, and a transverse crease in the mid philtral area. These findings are most consistent with the action of which of the following muscles?
A) Depressor septi nasi
B) Nasalis
C) Procerus
D) Risorius
E) Zygomaticus major
A

A) Depressor septi nasi

A deformity upon facial animation characterized by descent of the nasal tip, shortening of the upper lip, and a transverse crease in the mid philtral area may be created or accentuated by the action of the depressor septi nasi muscles. These are small, paired muscles located on each side of the nasal septum, which originate at the medial crura foot plates and insert either on the incisive fossa of the maxilla or into the fibers of the orbicularis oris muscle.

Physical examination upon facial animation should be part of the routine preoperative evaluation of the rhinoplasty patient. Those who present with the dynamic deformity as described may benefit from excision or transection of the depressor septi nasi muscles. Several surgical techniques have been described, as well as the use of botulinum toxin type A.

The nasalis muscle compresses the cartilaginous part of the nose and draws the ala toward the septum. Although this may generate some depression of the tip of the nose, it should not cause shortening of the upper lip.

The procerus muscle depresses the medial angle of the eyebrows, creating transverse rhytides over the bridge of the nose. The risorius muscles retract the angles of the mouth, as in a grinning expression. The zygomaticus major muscles draw the angles of the mouth posteriorly and superiorly, as in laughing. These muscles do not cause depression of the tip of the nose.

415
Q
During the period of mixed dentition, which of the following is the first permanent tooth to erupt?
A) Mandibular canine
B) Mandibular first molar
C) Mandibular first premolar
D) Maxillary central incisor
E) Maxillary lateral incisor
A

B) Mandibular first molar

The stage of mixed dentition is defined as the age range in which there are both deciduous (primary) and permanent (secondary) teeth erupted in the oral cavity at the same time. Normally, the mandibular and maxillary teeth erupt in a slightly different pattern. This usually occurs at age 6 to 7 years and is completed by age 11 to 12 years. In the maxilla, the order of eruption is as follows: first molar, central incisor, lateral incisor, first premolar, second premolar, canine, second molar, and third molar. In the mandible, the order is slightly different and is as follows: first molar, central incisor, lateral incisor, canine, first premolar, second premolar, and second molar. The permanent first molars erupt between ages 6 and 7 years, the central and lateral incisors erupt between ages 6 and 8 years, and the first premolars erupt between ages 8 and 9 years. The first tooth to erupt is the permanent mandibular first molar, which erupts first in a position posterior to the deciduous second molar. There are no premolars in deciduous teeth.

416
Q

A 22-year-old man comes to the emergency department after he sustained a machete laceration of the left cheek extending from the tragus through the midpoint of the upper lip. The wound is full thickness along the central third. Examination shows left upper lip droop and flattening of the associated nasolabial fold. Which of the following structures were most likely injured?
A) Lacrimal sac, mandibular branch of the facial nerve, and pterygoid muscle
B) Maxillary sinus, zygomatic branch of the facial nerve, and pterygoid muscle
C) Parotid duct, buccal branch of the facial nerve, and masseter muscle
D) Zygomatic arch, zygomatic branch of the facial nerve, and orbicular muscle

A

C) Parotid duct, buccal branch of the facial nerve, and masseter muscle

The middle third of a line drawn between the tragus and the middle of the upper lip defines the course of the parotid duct. The buccal and zygomatic branches of the facial nerve lie in close proximity to the parotid duct, which lies superficial to the masseter muscle. Deep penetrating trauma in this region is likely to injure all three of these structures. Evidence of injury to the zygomatic or buccal branch of the facial nerve with a central cheek laceration should raise concern for a parotid duct injury.

The lacrimal sac is outside of the described zone of injury, as is the mandibular branch of the facial nerve. The pterygoid muscle is deep to the mandible and would be outside of the described zone of injury.

The maxillary sinus and the zygomatic branch of the facial nerve could have been injured, but not in combination with the pterygoid muscle, which is outside the zone of injury.

Concomitant injury of the zygomatic arch and zygomatic branch of the facial nerve is possible, but because this injury was full thickness in the central third, the orbicularis muscle would not have been involved.

417
Q

A 4-year-old girl is brought to the office because of a congenital mucous-draining skin sinus located on the lower third of the neck, overlying the anterior border of the left sternocleidomastoid muscle. On physical examination, swallowing causes noticeable puckering of the sinus. Intraoperative probing shows that it communicates with the left tonsillar fossa. On surgical exploration, which of the following is the most likely ascending course of this sinus?
A) Deep to the hypoglossal nerve
B) Deep to the internal carotid artery
C) Superficial to the posterior belly of the digastric muscle
D) Superficial to the stylohyoid muscle
E) Superficial to the stylopharyngeal muscle

A

E) Superficial to the stylopharyngeal muscle

Surgical exploration is most likely to show the ascending course of this pharyngeal fistula to be superficial to the stylopharyngeal muscle.

Second pharyngeal cleft and pouch anomalies (including cysts, fistulas, and sinuses) account for 67 to 95% of the total anomalies of the pharyngeal apparatus. Cysts are the most common finding, occurring three times more often than fistulas. Fistulas usually present at birth. They derive from the ventral portion of the second pharyngeal cleft and pouch. The external opening is usually found along the anterior border of the sternocleidomastoid muscle, between the hyoid bone superiorly and the suprasternal notch inferiorly. Fistulas have a muscular coat, which is continuous superficially with the platysma and internally with the palatopharyngeal muscle. If this muscle coat is well developed, swallowing causes a pull on the fistulous opening, resulting in puckering.

The anatomical relations between a second pharyngeal cleft and pouch fistula and the surrounding cervical structures are dictated by the embryogenesis of the pharyngeal apparatus. As an anomaly of the second cleft and pouch, the fistula is expected to course deeply to the second arch structures and superficially to the structures derived from the third to sixth arches.

Therefore, the described fistula is expected to course deep to the stylohyoid muscle and posterior belly of the digastric muscle (derived from the second pharyngeal arch), and superficial to the internal carotid artery and stylopharyngeal muscle (derived from the third pharyngeal arch).

The hypoglossal nerve and associated infrahyoid muscles do not develop in the mesenchyme of the pharyngeal apparatus, instead being derived from occipital somites in the paraxial mesoderm. All pharyngeal anomalies derived from ectoderm (e.g., fistulas) will be found superficial to the hypoglossal nerve and the infrahyoid strap muscles.

Other cervical structures not mentioned in this scenario but which are relevant to the course of the described fistula include the external carotid artery (second pharyngeal arch) and the glossopharyngeal nerve (third pharyngeal arch). The expected fistula course is deep to the former and superficial to the latter.

418
Q
A 25-year-old man comes to the office for consultation regarding a 10-year history of gradual swelling of the right side of the face. Physical examination shows class I occlusion, normal interincisal distance, and smooth occlusal surfaces on the right molar teeth. The right cheek is enlarged when he clenches the teeth. CT scan shows a right masseter that is twice as large as the left one, and there is an outward curvature of the angle of the mandible. Which of the following is the most appropriate treatment?
A) Condylar reduction
B) Masseter resection
C) Orthodontics
D) Radiation therapy
E) Sagittal split osteotomy
A

B) Masseter resection

Benign masseteric hypertrophy may present as a bilateral or a unilateral condition. When unilateral, it is associated with repetitive unilateral clenching of the teeth. Both the masseter and temporalis muscles of the affected side show varying degrees of enlargement. When mild, medical management may be offered first and can include muscle relaxants, anxiolytics, antiepileptic drugs, and botulinum toxin type A. Surgical resection of a portion of the masseter and/or bone contouring are appropriate surgical procedures for correction of the resulting cosmetic deformity.

Unilateral masseteric hypertrophy must be distinguished from unilateral condylar hyperplasia, the latter consisting of the enlargement or overgrowth of the mandibular condyle. Condylar hyperplasia may also present with unilateral facial enlargement (type IB or type II), deviation of the mandibular midpoint toward the unaffected side, class III occlusion on the ipsilateral side, and a crossbite on the contralateral side. Condylar resection is the mainstay of treatment.

Condylar reduction is appropriate for cases of condylar dislocation. This condition can occur unilaterally in patients with a hypermobile or stretched temporomandibular joint, or in patients with dystonia (hyperfunction of the lateral pterygoid muscle). Condylar dislocation occurs suddenly, and causes pain and a class III occlusion on the involved side. Condylar reduction is performed with the aid of muscle relaxants. Eminence surgery may be necessary (eminectomy, eminoplasty).

Orthodontics are unnecessary for someone with bruxism, which is a typical feature of masseteric hypertrophy. However, a mouth guard is appropriate.

Radiation is inappropriate for benign masseteric hypertrophy. It does not correct the underlying cause or the deformity associated with this condition.

Sagittal split osteotomy is indicated in cases of class II or III malocclusion in which reduction or lengthening of the anteroposterior length of the mandible will achieve dental harmony. This procedure is not appropriate for class I occlusion or flaring of the mandibular angle.

419
Q
Which of the following cranial nerves is responsible for parasympathetic innervation to the parotid gland?
A) Trigeminal (V) nerve
B) Vestibulocochlear (VIII) nerve
C) Glossopharyngeal (IX) nerve
D) Vagus (X) nerve
E) Hypoglossal (XII) nerve
A

C) Glossopharyngeal (IX) nerve

The innervation of the parotid gland comes from parasympathetic fibers of the glossopharyngeal nerve (cranial nerve IX). It also receives taste sensation (afferent) from the posterior one third of the tongue.

The maxillary nerve of cranial nerve V (V2) is a sensory nerve and receives sensation from the mid face.

Parasympathetic fibers (efferent) innervate the submandibular and sublingual glands via the chorda tympani. Afferent fibers, via the chorda tympani, send taste sensation of the anterior two thirds of the tongue.

The vestibulocochlear nerve (cranial nerve VIII) supplies sound and equilibrium to the brain.

The Arnold nerve, also called the auricular branch of the vagus nerve (cranial nerve X), innervates the external acoustic meatus. Stimulation of the Arnold nerve can lead to reflex coughing (Arnold reflex).

420
Q
A 35-year-old woman is evaluated because of numbness of the upper helical rim of the left ear 30 days after she underwent neurosurgical decompression to treat facial pain. Which of the following nerves was most likely injured?
A) Auriculotemporal
B) Glossopharyngeal
C) Great auricular
D) Lesser occipital
E) Vagus
A

A) Auriculotemporal

Knowledge of the innervation of the external ear is critical to the understanding of its embryologic development, as well as in the delivery of adequate local anesthesia for minor surgical procedures. Sensation to the external ear is derived from several cranial and extracranial nerve branches. The great auricular (C2 to C3) and lesser occipital (C2) are cranial nerves which innervate the posterior aspect of the auricle and lobule. While the distribution is variable, in most cases the lesser occipital supplies the superior ear and mastoid region while the great auricular nerve supplies the inferior ear and a portion of the preauricular area. The anterior surface of the ear, including the helix, scapha, and concha, is supplied by the auriculotemporal nerve (V3 trigeminal) and is most likely to be injured in a microvascular decompression for the treatment of trigeminal neuralgia. Branches of the vagus (X) and glossopharyngeal (IX) nerve innervate the external auditory meatus.

The innervation to the external ear follows its embryologic branchial arch origins with the great auricular nerve innervating first branchial arch structures and the auriculotemporal nerve innervating second branchial arch structures. An auriculotemporal nerve block provides anesthesia to the helix and tragus and is approached by injecting 2 to 4 mL of anesthesia superiorly and anteriorly to the tragus. The great auricular nerves and lesser occipital nerves are blocked by injecting 2 to 4 mL of anesthetic to the posterior sulcus from the inferior aspect of the earlobe. This will provide anesthesia to the earlobe and lateral helix.

421
Q
A 50-year-old man undergoes wide local excision and bilateral selective cervical lymphadenectomy because of a 6-month history of invasive squamous cell carcinoma of the anterior floor of the mouth. Free tissue transfer using an anterolateral thigh free flap, including harvest of the lateral femoral cutaneous nerve, reconstructs the ventral glossectomy and floor-of-mouth defect. Which of the following is the most likely recipient nerve for functional sensory recovery of the free flap in this patient?
A ) Cervical branch of facial
B ) Great auricular
C ) Hypoglossal
D ) Inferior alveolar
E ) Lingual
A

E ) Lingual

Oral mucosal sensation is important in many stomatognathic functions. Mastication, oral hygiene, phonation, and swallowing can all influence patient quality of life. A proportionally worsening functional impact with an increasing area of anesthesia has been noted. Therefore, restoration of sensibility should be one of the important components of the functional rehabilitation of glossectomy defects.

Although spontaneous reinnervation does occur in noninnervated flaps, it takes a longer period of time to develop, and it may not restore adequate functional sensation, nor does it provide useful tactile sensation or two-point discrimination. These sensory modalities are important in a patient’s ability to handle oral secretions and food boluses.

Microsurgical anastomoses of the lateral femoral cutaneous nerve is most commonly performed to the lingual nerve stump left after tumor extirpation.

The cervical branch is the lowest division of facial motor nerve and would not provide sensory recovery to the freely transferred thigh tissue.

The great auricular nerve provides sensation to the earlobe and can serve as an ideal donor for segmental nerve grafting, if required. Bioprosthetic conduits (nerve tubes) or vein grafts have also been described for this purpose.

The hypoglossal nerve is commonly identified and preserved in selective cervical lymphadenectomy, and, if divided, causes a motor paralysis of the ipsilateral tongue.

The inferior alveolar nerve is part of the trigeminal system (V3) and is not divided during a floor-of-mouth resection and/or glossectomy, unless a segmental mandibulectomy is also performed.

422
Q

Which of the following best describes the anatomical location of the nerve that supplies sensation to the frontoparietal scalp at the level of the forehead?
A ) In the subcutaneous fat
B ) Superficial to the frontalis muscle
C ) Superficial to the galea aponeurosis
D ) Superficial to the periosteum
E ) Through the medial corrugator supercilii muscle

A

D ) Superficial to the periosteum

The supraorbital nerve (SON) supplies sensation to the forehead skin (paramedian) and anterior scalp as well as the frontoparietal scalp. The latter scalp is supplied by the deep division of the nerve, whereas the rest is supplied by its superficial division. Medially passing through the corrugator supercilii muscle is the supratrochlear nerve, which supplies the medial skin of the forehead. The deep division of the SON travels initially (just medial to the superotemporal line) along the temporal periosteum and then more cephalad pierces the deep galea plane and enters the galea fat pat. This information is the key to avoid injury when performing a forehead lift. Also, when planning a bicoronal incision for craniofacial surgery, one can use this information to preserve the sensation of the related scalp. The superficial branch of the SON, on the other hand, travels superficial to the frontalis muscle in the paramedian forehead area.